You are on page 1of 109

Algebra through practice

Book 3: Groups, rings and fields


Algebra through practice
A collection of problems in algebra with solutions

Book 3
Groups, rings and
fields
T.S.BLYTH o E.F.ROBERTSON
University ofSt Andrews

The right of the


University of Cambridge
to print and sell
all manner of books
was granted by
Henry VIII in 1534.
The University has printed
and published continuously
since 1584.

CAMBRIDGE UNIVERSITY PRESS


Cambridge
London New York New Rochelle
Melbourne Sydney
CAMBRIDGE UNIVERSITY PRESS
Cambridge, New York, Melbourne, Madrid, Cape Town, Singapore, Sao Paulo

Cambridge University Press

The Edinburgh Building, Cambridge CB2 8RU, UK

Published in the United States of America by Cambridge University Press, New York

www. Cambridge. org


Information on this title: www.cambridge.org/9780521272889
© Cambridge University Press 1984

This publication is in copyright. Subject to statutory exception


and to the provisions of relevant collective licensing agreements,
no reproduction of any part may take place without the written
permission of Cambridge University Press.

First published 1984


Re-issued in this digitally printed version 2008

A catalogue record for this publication is available from the British Library

Library of Congress Catalogue Card Number: 83-24013

ISBN 978-0-521-27288-9 paperback


Contents

Preface vii
Background reference material ix
1: Groups 1
2: Rings and fields 19
Solutions to Chapter 1 32
Solutions to Chapter 2 63
Test paper 1 88
Test paper 2 90
Test paper 3 92
Test paper 4 94
Preface

The aim of this series of problem-solvers is to provide a selection of worked


examples in algebra designed to supplement undergraduate algebra courses.
We have attempted, mainly with the average student in mind, to produce
a varied selection of exercises while incorporating a few of a more challenging
nature. Although complete solutions are included, it is intended that these
should be consulted by readers only after they have attempted the questions.
In this way, it is hoped that the student will gain confidence in his or her
approach to the art of problem-solving which, after all, is what mathematics
is all about.
The problems, although arranged in chapters, have not been 'graded'
within each chapter so that, if readers cannot do problem n this should not
discourage them from attempting problem n 4-1. A great many of the ideas
involved in these problems have been used in examination papers of one sort
or another. Some test papers (without solutions) are included at the end of
each book; these contain questions based on the topics covered.

TSB, EFR
St Andrews
Background reference material

Courses on abstract algebra can be very different in style and content.


Likewise, textbooks recommended for these courses can vary enormously,
not only in notation and exposition but also in their level of sophistication.
Here is a list of some major texts that are widely used and to which the
reader may refer for background material. The subject matter of these texts
covers all six of the present books, and in some cases a great deal more. For
the convenience of the reader there is given overleaf an indication of which
parts of which of these texts is most relevant to the appropriate chapters of
this book.

[1] I. T. Adamson, Introduction to Field Theory, Cambridge University Press,


1982.
[2] F. Ayres, Jr, Modern Algebra, Schaum's Outline Series, McGraw-Hill, 1965.
[3] D. Burton, A First Course in Rings and Ideals, Addison-Wesley, 1970.
[4] P. M. Cohn, Algebra, Vol. I, Wiley, 1982.
[5] D. T. Finkbeiner II, Introduction to Matrices and Linear Transformations,
Freeman, 1978.
[6] R. Godement, Algebra, Kershaw, 1983.
[7] J. A. Green, Sets and Groups, Routledge and Kegan Paul, 1965.
[8] I. N. Herstein, Topics in Algebra, Wiley, 1977.
[9] K. Hoffman and R. Kunze, Linear Algebra, Prentice Hall, 1971.
[10] S. Lang, Introduction to Linear Algebra, Addison-Wesley, 1970.
[11] S. Lipschutz, Linear Algebra, Schaum's Outline Series, McGraw-Hill, 1974.
[12] I. D. Macdonald, The Theory of Groups, Oxford University Press, 1968.
[13] S. MacLane and G. Bkkhoff, Algebra, Macmillan, 1968.
[14] N. H. McCoy, Introduction to Modern Algebra, Allyn and Bacon, 1975.
[15] J. J. Rotman, The Theory of Groups: An Introduction, Allyn and Bacon,
1973.
Book 3: Groups, rings and fields

[16] I. Stewart, Galois Theory, Chapman and Hall, 1973.


[17] I. Stewart and D. Tall, The Foundations of Mathematics, Oxford University
Press, 1977.

References useful to Book 3


1: Groups [2, Chapter 9], [6, Chapter 7], [7, Chapters 4, 5, 6],
[8, Chapter 2], [14, Chapters 7, 8].
2: Rings and fields [2, Chapters 10,11,12], [6, Chapter 8], [8, Chapter 3],
[14, Chapters 2, 3,9,10].

In [2, 7,14] the authors write mappings on the right and, as a consequence
multiply permutations accordingly. In contrast, in [8] most mappings are
written on the left, but permutations are written and multiplied as mappings
on the right. Also in [8] an integral domain is not required to have a 1. In [6]
integral domains are not required to be commutative, and all rings have a 1.
1: Groups

Abstract algebra is basically a study of sets with binary operations. A binary


operation (or law of composition) on a set E is a mapping f:ExE->E
described variously by (x, y) ->x * y, (x, y) ->x + y, (x, y) ->xy, etc. When
E is finite it is sometimes convenient to represent a binary operation on E by
means of a Cayley table, the interpretation of which is that x,- * x;- appears at
the intersection of the /th row and the/th column (Fig. 1.1). A binary operation

Fig.1.1
• x
i

x
1
i

* on E is associative if (Vx, y9 zGE)x * (y * z) = (x * y) * z. A group is


a set G on which there is defined an associative law of composition * such that
(a) there is an identity element (i.e. an element e such that
(Vx E G)e * x = x = x * e);
(b) every element of G has an inverse (i.e. for every xGG there
exists jy E G such that x * y — e =y * x).
Book 3: Groups, rings and fields

When the law of composition is written as addition (respectively multiplica-


tion) we denote the identity element by 0 (respectively 1) and the inverse
of x £ G by —x (respectively x" 1 ). Elements x, y of a group G are said to
commute if xy = yx, and the group is said to be abelian if every pair of
elements commute.
In studying algebraic structures there are two important notions to con-
sider. The first is that of a substructure, and the other is that of a structure-
preserving mapping from one such structure to another.
A subgroup of a group G is a non-empty subset H with the property that
H is closed (or stable) under the operation of G (i.e. x, yEH=*xyGH)
and is also a group under the law of composition that is thus inherited from
G. To show that a non-empty subset H is a subgroup of G it suffices to prove
that H satisfies the property

In additive notation, this becomes


x,yGH =>x— y£H.
Alternatively, one can check that H is stable and that y GH^y"1 £ / / . In
order to prove that a particular set H together with a given law of composition
forms a group, it is often best to show that H is a subgroup of a larger set
which is known to be a group. For example, the set 2Z of even integers is
a group under addition, being a subgroup of the additive group Z. If G is
a group and S is a non-empty subset of G then the smallest subgroup of
G that contains S is denoted by (S) and consists of all products of powers of
elements of S. In particular, if S = {g} then the subgroup (S) = (g) is said to
be cyclic. It is given by:
<g)={gi\iet}
where g° = 1. Note that all the powers of g need not be distinct: it is possible
to have g™ =gn with mi=n, so that gm~n — 1. The smallest positive integer
k (when it exists) such thatgfc = 1 is called the order of g in G. When no such
k exists, G is said to have infinite order. This should not be confused with
what is called the order of the group G, namely the number |G| of elements
in G. Nevertheless, note that \(g)\ is equal to the order of g. For every positive
integer n there is a cyclic group of order n. This is denoted by Cn. All sub-
groups of Cn are of the form Cm where m divides n. The celebrated theorem
of Lagrange states that for a finite group the order of every subgroup divides
the order of the group; i.e. if His a subgroup of G then \H\ is a factor of | G\.
/ : Groups

The usual proof of Lagrange's theorem uses the notion of a coset. If H is


a subgroup of G then the right cosets (respectively left cosets) of / / i n G are
the sets Hx = {hx\hEH} (respectively xH={xh \ h EH}). The set of right
(respectively left) cosets of H in G forms a partition of G and the number of
cosets is called the index of H in G. A subgroup H of G is said to be normal
in G if (VJC E G)XH = Hx. When H is normal, the set of cosets forms a group
under the law of composition given by xH.yH — xyH. This group is called
the quotient group of G by H and is written G/H.
The notion of a normal subgroup is intimately related to that of a structure-
preserving mapping. If G, H are groups then a mapping / : G ->// is called
a group morphism if (Vx, 7 EG)f(xy) =f(x)f(y). Group morphisms are
very often called homomorphisms. A bijective group morphism is called an
isomorphism. There are two important subsets associated with every group
morphism/: G->//, namely the kernel and image of/, defined respectively by
Ker/ = {xEG | /(*) = 1};
lmf={f(x)\xEG}.
For every group morphism / : G ->• //, Ker / is a normal subgroup of G. Con-
versely, every normal subgroup is the kernel of some group morphism. The
first isomorphism theorem for groups states that if / : G -> H is a group
morphism then I m / i s isomorphic to G/Ker/; in symbols, I m / ^ G/Ker/
We assume that the reader is familiar with the notion of a permutation
(simply a bijection on a finite set) and the notation
1 2 3 ... /

as well as the cycle notation for permutations. Here we treat permutations


precisely as mappings, so that multiplication of permutations is their compo-
sition as mappings. For example,
/I 2 3W1 2 3 \ _ / l 2 3\
\3 2 l / \ 2 1 3 / \ 2 3 1 / '
or, in cycle notation,
(13)(1 2) = ( 1 2 3).
The set of all permutations on a set of n elements (which is usually chosen
to be {1, 2, ...,n}) is a multiplicative group denoted by Sn.
As an illustration of the first isomorphism theorem, consider the mapping
& : S 3 -> S3 defined by
Book 3: Groups, rings and fields

(1) if x is even;
1(12) if* is odd.
# is a group morphism and S2 = Im #^*S 3 /Ker # where Ker # = { ( 1 ) ,
(1 2 3), (3 2 1)} is the cyclic subgroup of S3 generated by {(1 2 3)}.
1.1 Show that the prescription
[a][b]=[ab]
defines a binary operation on the set Xn of congruence classes modulo n.
Show also that the prescription
[a]+[b]=[a + b]
defines a binary operation on Z w .
Find [a], [b] G Z 6 with [a] =£ [0] so that the equation
[a][x]=[b]
has (a) no solution, (b) exactly one solution for [x] G Z6- Is it possible to
find [a], [b] G Z 6 so that the equation has more than one solution?
1.2 LetR be the equivalence relation defined on Z by
xRy if and only if x2 = y2 (mod 6).
Let S denote the set of equivalence classes. Show that [•*]#[;>]# = [xy]R
defines a binary operation on S and compile the associated Cayley table.
Does [X]R 4- [y]ji = [x + y]ji define a binary operation on SI
1.3 Let P = {p G Z | p is a prime and p < 13}. Define a binary operation * on
Pby
p * q = the greatest prime divisor of p + q — 2.
Construct the Cayley table for * and show that P has an identity with respect
to *. Does every element of P have an inverse? Is * associative?
1.4 A binary operation * is defined o n J £ ' = I R x l R x l R b y
(x,y, z) * (*',/, z') = (xx\ ay'+yx', az' + zx')
where a = x + y 4- z. Show that * is associative and that E contains an
identity with respect to *. Show also that (x, y, z) has an inverse if and
only if x and a. are both non-zero. Find the inverse of (x, y, z) when it exists.
1.5 For each of the following sets, determine whether the given operation is
associative. Does the set contain an identity for the given operation? If it
has an identity does every element have an inverse? Is the set a group under
the given operation?
1: Groups

(a) The even integers under addition.


(b) The rationals under subtraction.
(c) The rationals under multiplication.
(d) The positive rationals under multiplication.
(e) Z 6 \ {[0]} under multiplication.
(/) Z 5 under multiplication.
(g) The cube roots of 1 under multiplication.

1.6 Which of the following sets are groups under the operation * given?
(a) Positive integers with a * b = max {a, b).
(b) Z with a * b = min {a, b).
(c) \Rwitha*b=a + b-ab.
(d) Positive integers with a * b = max {a, b) — min {a, b).
(e) Z x Z with (a, b) * (c, d) = (a + c, b + d).
( / ) IR x IR with (a, b) * (c,rf)= (ac + M, ad 4- bd).
(g) (IR x IR) \ {(0, 0)} with (a, b) * (c, d) = (flc - M, ad +
1.7 Let 7" be the set of matrices of the form
1x
f
where JC, y G IR. Prove that T is a group under matrix addition, and that
T\ {0} is a group under matrix multiplication.
1.8 Let G be the set of mappings / : IR -HR of the form f(x) = ax + b with
a =£ 0. Prove that G forms a group under composition of mappings.
1.9 Define a binary operation * on IR* = IR \ {0} by
ab
a *b= a
if a < 0.
I
Is IR* a group under *?
1.10 Determine the group of symmetries of each of the figures in Fig. 1.2.

Fig.1.2
Book 3: Groups, rings and fields

1.11 Prove that the group of symmetries of a regular w-sided polygon (with n > 3)
has 2n elements and describe them. Prove that none of these dihedral groups
is abelian.
1.12 Consider the infinite pattern
. . .VVVVV...
Clearly the symmetries of this consist of
(a) translations a number of units in either direction;
(b) reflections in a vertical line either through the centre of any
symbol or between two symbols.
Let a be the translation one unit to the right and let b be the reflection in
a vertical line through the centre of a fixed symbol. Show that the group of
symmetries is generated by a and b subject to the relations b2 = 1, ba = a~ib.
There are seven linear' patterns with different symmetry groups. Can you
find them all?
1.13 Let G be the group of rotations of a cube. Show that \G\ = 24. Find eight
different non-identity elements satisfying x3 = 1 which fix a corner. Find
nine non-identity elements satisfying x 4 = 1 which take a given face to itself.
Find six non-identity elements satisfying x2 = 1 which take a given edge to
itself. Check that each of the 24 elements has now been identified.
Do the same for the group of rotations of an octahedron (the regular solid
with six vertices and eight triangular faces).
(Hint: This can be deduced easily from the group of rotations of a cube!)
1.14 The following is part of the Cayley table of a finite group. Fill in the missing
entries:
e a b X y z
e e a b X y z
a a b e y
b b
X X z a
y y
z z
1.15 Use Lagrange's theorem to show that if G is a group with \G\ = n then
g" = 1 for every g G G. Deduce that if p is a prime then aP = tf(mod p) for any
1: Groups

integer a. Hence find the remainder on dividing


(a) 7 1 0 0 b y l l ;
(b) 9 3 7 b y l 3 .
1.16 Let G be a group and let a, b £ G be such that ab = bak where k is a fixed
positive integer. Prove by induction that, for all positive integers n,
(a) anb = bakn;
(b) abn = bnakH.
Suppose now that b is of order 2. Prove that (ban)2 commutes with
both a and b.
1.17 Let G be a group and let a, b €= G be such that ab =£ ba. Prove that the elements
1, a, b, ab, ba are distinct. Show further that either a2 = 1 or a2 & {1, a, b,
ab, ba}. In the case where a2 = 1 show that aba £ { 1 , a, b, ab, ba}. Deduce
that every non-abelian group contains at least six elements.
Let F be the set of six functions from IR \ {0, 1} to itself given by
1 JC —1
e(x) = x, p(x) = , q(x) = ,
1—x x
1 X
a(x)=-, b{x)=\-x, c(x) = -.
x x—1
Construct the Cayley table for F under o and show that F is a non-abelian
group.
1.18 (a) Let a and b be elements of a group G which commute. Prove that
a and b'1 commute. Prove also that if x is any element of G thenxax'1 and
xbx~l commute.
(b) Prove that a group G is abelian if and only if (ab)'1 = a'lb~l for all
a,bGG.
(c) Prove that if G is a group such that (ab)2 = a2b2 for alla,b E G then
G is abelian.
(<i) Let G be a group and suppose that a2 = 1 for every aGG. Prove that
G is abelian.
(e) If Gis a group a n d x , y GG prove by induction that, for every positive
integer n, (x~lyx)n = x~lynx.
( / ) Let G be a group and suppose that a, b EG are such that b6 = 1 and
#Z> = &4tf. Prove that b3 = 1 and that #& = Zw.
1.19 Let G be the multiplicative group of non-singular 2 x 2 matrices with rational
entries. What are the orders of the following elements of G?
Book 3: Groups, rings and fields

li i J ' 1-3 - i J '


2 2- Li oJ"
Find an element of order 2 in G.
Prove by induction that if
M 11 r l n~\
Lo iJ Lo iJ
for every nG IN. Deduce that A does not have finite order. Is G a finite
group? Find a, b, cGG such that a and b commute, b and c commute but
a and c do not commute.
1.20 Which of the following groups are abelian? Which are cyclic? Which are finite?
ffl nl \
(a) G = { | n G Z J under multiplication.

(b) The non-zero elements of Z n under multiplication.


(c) The group of bijections of E = {1, 2, 3} to itself under compo-
sition of mappings.
(d) The group of bijections of E— {1, 2} to itself under composition
of mappings.
(e) The even integers under addition.
( / ) IR under addition.
1.21 On the set G = IR \ {0} x IR define a law of composition by
(a, b)(c, d) = (ac, be 4- d).
Prove that G is a non-abelian group.
Determine which of the following subsets of G are subgroups.
H = {(a, k(a ~ 1)) \a ^ 0} where k G IR is fixed;
K={(a,0)\a>0}',
L = {(a, nan) \a ^ 0} where w G IN is fixed;
Jlf={(l,6)|6GIR}.
Show that G contains an infinite number of elements of order 2. Does
G contain elements of order 3?
1.22 (a) Let G denote the group of non-zero rationals under multiplication.
Let
( l + 2« \
H={2n\n€X}, K={ \n9mez.\.
l l + 2m I
1: Groups

Are H and K subgroups of Gl


(b) Show that # = { [ 0 ] , [4], [8], [12]} is a subgroup of Z i 6 under
addition.
(c) Find all the subgroups of the group in question 1.14.
1.23 Let S = {(x,y) | JC, y G !R, x & 0, x + y =£ 0}. Prove that the binary operation
defined on S by

makes S into a group.


Show that P = {(1, y) | .y > —1} is a subgroup of S.
1.24 Prove that

G= \\a9b9c9dGZ9ad-bc=l\
\lc d\ )
is a group under multiplication. Let HN be the subset of G consisting of
[a bl
those matrices in G with
Ic d\
a = d=l (mod TV), b = c = 0(mod N),
N being a fixed positive integer. Prove that HN is a subgroup of G.
If S = {HN | 2 < i V < 12} is ordered by a set inclusion, draw the Hasse
diagram for S.
1.25 Let p be an odd prime. A non-zero integer a is said to be a quadratic residue
of p if there exists an integer x such that x2 = #(mod p). Prove that
(a) the quadratic residues of p form a subgroup Q of the group of non-
zero integers mod p under multiplication;

(c) if q GQ,n&Q thennq is not a quadratic residue;


(d) ifm and « are not quadratic residues then mn is a quadratic residue;
(e) if 0 is a quadratic residue then a^p~^n = 1 (mod p).
1.26 Let M = {M(d) \ & G IR} where

[cos # —sin #1
sin # cos # J
Prove that M is an abelian group. Show that M contains cyclic subgroups of
every finite order. Does M contain infinite cyclic subgroups?
1.27 Let G = (g) be a cyclic group of finite order n generated by the element
Book 3: Groups, rings and fields

g. What are the subgroups of G? Prove that the other generators of G are
the elements g^ where hcf {r, n} = 1.
For the group Zi 8 list all the subgroups and find all the generators in each
subgroup. Draw the subgroup Hasse diagram.
1.28 Prove that if G is a group and H, K are subgroups of G then H n K is a sub-
group of G. Suppose now that G is finite and that hcf {\H\, \K\} = 1. Prove

Find mZ Pi riZ. in the additive group Z. Deduce that no two non-trivial


subgroups of Z can intersect in the trivial subgroup.
1.29 Suppose that G is a group. For everya EG define
C(a) = {xEG\ax = xa}.
Prove that C(a) is a subgroup of G.
Now suppose that H is a subgroup of G. Define
C(H) = {x E G I (VaeH)ax = xa}.
Prove that C(H) is a subgroup of G.
Give examples of situations where
(a) C(H) = H;
(b)
(c)
1.30 If G is an abelian group and a,b EG are distinct elements of order 2, show
that ah has order 2. Prove that {1, a, b,ab} forms a subgroup of G that is not
cyclic.
Consider the set
G = {[n] | n an odd integer},
where [n] denotes the congruence class of n modulo 2k+1 and k is a fixed
integer greater than 1., Given that G is a group under the usual multiplication
of congruence classes show, by considering the elements [2^ — 1] and [2k + 1],
that G is not cyclic.
1.31 Let (D be the additive group of rationals. If
Pi P2
= — G (Q and r2 = — £ (Q
prove that
/ 1 v

10
1: Groups

Extend this result to any finite collection of elements of (D and deduce that
if>!,...,r n G (Q then {ru . . . , rn) is cyclic.
Show that if Hn is the subgroup of (D generated by \\n\ then

with each Hn cyclic. Show, moreover, that each of these inclusions is strict.
Prove that (D = Uw>i #„, so that (D is a union of cyclic groups.
Prove finally that (Q is not cyclic.
1.32 Let G be a group and let H be a subgroup of G. Prove that the following
conditions on H are equivalent:
(a) (Vx GG)xH = Hx (i.e. # is normal);
(6) (\/xeG)x~1HxCH;
(c) (\/xeG)x~1Hx=H;
(d) (yx E G)(yh GH)x~yhx GH;
(e) (\/xeG)(VheH)(3h'GH)xh = tix;
if) (Vx G G)(VA GH)h-lx'xhxeK
1.33 Let TV be a subgroup of a group G. Prove that the following conditions are
equivalent:
(1) TV is normal in G;
(2) (Wx e G)(yn e N)xN = nxTV.
Are the following conditions equivalent?
(a) N is normal in G;
(b) (yx G G)(V/i GN)xnx EN.
1.34 If a subgroup # has index 2 in a group G prove that H is normal in G.
\a 0 ]
Let ^4 be the set of matrices of the form and let B be the set

of matrices of the form,f°"}


Vb OJ
where a, b are non-zero real numbers. Prove

that G = A U 2? is a group under matrix multiplication. Prove also that A is


a subgroup of G and that .4, 5 are the cosets of A in G. Deduce that ^4 is
normal in G.

Show that the set of matrices of the form is a normal subgroup


IS

Lo l J
of A. Deduce that 'a normal subgroup of a normal subgroup is not necessarily
normal'.

11
Book 3: Groups, rings and fields

1.35 Consider the set G of matrices of the form


1 a b
0 1 c
.0 0 1.
where a, b, cE.'Z.. Show that G is a group under matrix multiplication.
Prove that the subset H of matrices of the form
1 0) aa
0 1i 0 ,
.0 0) 1.
where a E Z , is a normal subgroup of G. Prove also that the set K of
matrices of the form
"l a 0"
0 1 0 ,
.0 0 1.
where a E Z, is a subgroup of G. Is K a normal subgroup?

1.36 . Let G be a group with the property that there exists a positive integer n such
that (Vx,j> G G)(;cy)w = *"/*• Prove that the subsets
Gn = {xn \xGG} and Gn = { x G G | / = l}
are normal subgroups of G and that Gn = i
1.37 (a) Write the element

c 2 3 4 5 6 7 8 9\
'469725 1 3/
of S9 as a product of disjoint cycles.
(b) Express each of the following permutations as products of disjoint
cycles:
/ I 2 3 4 5 6 7 8\ /I 2 3 4 5 6 7
\ 8 2 6 3 7 4 5 1/ \3 6 4 1 8 2 5
/ 1 2 3 4 5 6 7
\3 1 4 7 2 5 8 6i
0 ) Write (143)(534)(137) as a product of disjoint cycles.
1.38 Do the elements (12)(34) and (13)(24) of 5 4 commute? Do the elements
(12)(24) and (13)(34) of S4 commute?

12
1: Groups

1.39 Prove that the order of a cycle of length r in Sn is r.


If a permutation a is a product of disjoint cycles of lengths r ands, prove
that the order of o is the lowest common multiple of r and s.
Express the permutation
_/l 2 3 4 5 6 7 8 9\
\3 7 8 9 4 5 2 1 6/
as a product of disjoint cycles. Hence determine the order of a. Compute
a 1000 .
1.40 (a) Find aTlba when a = (135)(12) and b = (1579). What is the order of
aTlba1 Do the same for a = (579) and b = (123).
(ft) Compute rar" 1 where r = (135)(24) and a = (12)(3456).
1.41 If a is a permutation of 1, 2 , . . . , n and if (kik2 • • • kr) is a cycle, prove that
a(fc!*2 . . . fc^a"1 = (o(ki)o(k2) • • • a(* r )).
Deduce that {(1),(12)(34), (13)(24), (14)(23)} is a normal subgroup of S4.
1.42 Let Gi, G2 be groups and define a binary operation on Gi x G2 by

Show that this makes G\ x G2 into a group. Prove that H1 = Gix {1} and
H2 = {1} x G2 are normal subgroups of the cartesian product group G1xG2.
Show that if ht E: Hi and /*2 £ # 2 then hi and /z2 commute.
1.43 Which of the following are group morphisms? Find the image and kernel of
those that are morphisms.
(a) / : Z 1 2 - + z 1 2 , / ( M i 2 ) = [ * + l]i2;
(ft)/:C 1 2 -*C 1 2 , f(g)=g3;
(c) f: Z -* Z 2 x Z 4 , / W = (M 2 , [x]4);

(e) / : C 2 x C 3 ->5 3 ,
(f) f: Sn~*Sn+i> / 0 0 is ^ e permutation given by i ~+n(i) for
/ < « and « 4-1 -> « + 1.
1.44 Find all the morphisms from the additive group Z to the additive group (H
1.45 Prove that the set G of matrices of the form,
n-n -Hi
I n 1+nJ'
where n E Z, is an abelian group under multiplication. Show that G ^ Z.

13
Book 3: Groups, rings and fields

Show also that the set G\ of matrices of the form


n-2n n 1
L -An 1 + In J '
where n G Z, is a group. Is Gi ^ G?
1.46 Let Z 2 be the integers modulo 2 and let G be the group of matrices of
the form

[
c d\
a b~\

where the operation is usual matrix multiplication with the entries reduced
modulo 2. If T is the set consisting of the matrices

and if
mim
Li oJ
show that X ->AX describes a permutation on T. Hence show that G^S^.
1.47 Show that the group Z[X] under addition is isomorphic to the group (Q+ of
positive rationals under multiplication.
(Hint: Consider # : Z[X] -> (D+ given by
a0 + atX + • • • + anXn - ^ g y j 1 .. . pann
where p0 < px < •• • < pn are the first n + 1 primes.)
1.48 Let # be a fixed real number and let
0 1 -sin#"
-1 0 cos #
—sin # cos # 0
3
Show that ,4 = 0.
Givenx G IR, define the 3 x 3 matrix Ax by

where / 3 is the 3 x 3 identity matrix. Prove that, under matrix multiplication,


G = {Ax | x G IR} is an abelian group which is isomorphic to the additive
group IR.

1.49 Show that G = {(a, b) G IR x IR | b -=h 0} forms a group under the operation

14
1: Groups

defined by

Show that G is not abelian. Show also that the subsets


H={(fi,b)eG\a = 0},
K={(a,b)GG\b>0},
L= {(aib)GG\b=l}
are subgroups of G. Which of these subgroups are abelian?
Show that H O K is a group isomorphic to the multiplicative group IR+ of
positive reals.
Find all the elements of order 2 in G.
1.50 On the set G = Z x Z define a binary operation by
(a,b)(c,d) = (a + (-lfc,b+d).
Prove that G is a non-abelian group.
Let H={(a, b)<EG\b = 0} and K={(a, b)GG\a = 0}. Prove that
H and K are subgroups of G and determine whether or not they are normal.
If / : G -> G is given by /(#, Z>) = (0, b) prove that / is a group morphism
and find its image and kernel. Hence prove that the groups G/H and K are
isomorphic.
1.51 Let A be an abelian group and let E= {1, —1}. Define a binary operation
on G = A xEby
(a,e)(b,8) = (ab€,e8).
Prove that G is a group. Give examples where G is (0) abelian, (b) non-abelian.
Show that N= {(a, 1) \aEA} is a normal subgroup of G. Prove that
N^A and identify the group G/N.
Show that, with a suitable choice for the group A, the group G is iso-
morphic to the group in question 1.9.
1.52 Let G be an abelian group and let x, y G G. Prove that (xy)n = xnyn for all
positive integers n, and deduce that the subset H consisting of all the elements
of finite order is a subgroup of G.
Suppose now that G is the multiplicative group {zEC | \z\ — 1}. Prove
that in this case H = {ea7d | a G CD}.
If e ^ = e ^ show that

7T 7T

15
Book 3: Groups, rings and fields

Deduce that the prescription

/ ( e w ) = -+<Q

defines a mapping / : G -+ IR/(D. Prove that / is a group morphism and hence


show that G/H^ IR/CD.
1.53 Let G be the subgroup of the symmetric group S 4 generated by the permuta-
tions (12)(34) and (123). Show that the subgroup H of G generated by
(12)(34) and (13)(24) is a normal subgroup such that G/H is the cyclic
group of order 3.
Show that the subgroup K of H generated by (12)(34) is normal in # but
not normal in G.
1.54 Let G be the subgroup of the symmetric group S5 generated by (12345) and
(25)(34). Show that the subgroup H of G generated by (12345) is a normal
subgroup such that G/H is cyclic. Find the order of G/H and hence deter-
mine the order of G. Show that G is isomorphic to the symmetry group of
the regular pentagon.

1.55 Let m and n be integers with m a factor of n. Show that riZ. is a sub-
group of mZ and that mZ/^Z is a subgroup of Z/rcZ. Show that the
prescription
f(a + n~Z) = a + mX
defines a mapping Z/wZ -> Z/mZ. Show also that / is a group morphism
and find its kernel. Hence show that
Z/wZ Z
~~ raZ '
1.56 Let X be a set. Show that P(X) is a group under the binary operation A of
symmetric difference. Prove that this group is abelian and that every element
has order 2.
If X = {1, 2} show that P(X) s C2 x C2.
1.57 Let H be a subgroup of a finite group G. Let S be the set of left cosets of
HinG. If g G G show that the mapping $g: S->S given by
$g(aH) = gaH
is a bijection. Use this to define a group morphism # : G->Sk wherefc= |S|.
Prove that Ker $CH. Show, moreover, that Ker # is the largest normal

16
1: Groups

subgroup of G which is a subgroup of H. Prove that if |G| does not divide


kl then H contains a non-trivial normal subgroup of G.
Finally, show that if H is a subgroup of order 11 in a group G of order
99 then H is normal in G.
1.58 Each of the following groups has order 8:
(a) P(X) under A where X = {1, 2, 3};
(P) Z 8 ;
(c) the group of symmetries of a square;
(d) {1, 2, 4, 7, 8, 11, 13, 14} under multiplication modulo 15.
Show that no two are isomorphic.
1.59 Show that the set of isomorphisms from a group G to itself forms a group
under composition of mappings (the automorphism group Aut G of G).
Prove that
(a) AutZ^C 2 ;
(b) Aut Z w is isomorphic to the multiplicative group of integers
that are coprime to n\
(c)
1.60 Let Q be the quaternion group {±1, ±i, ±j, ±k} of order 8. Let H= Q x Q,
let Z* = {(1, 1), ( - 1 , - 1 ) } and let G = H/Z*.
Prove that the centre Z(G) of G has order 2. Show also that G/Z(G) is
abelian. Deduce that it is possible to write G as a cartesian product group
Gi x G2 only if G1 or G2 is the trivial group.
1.61 Let S be the set of real 2 x 2 matrices X such that X +1 2 is invertible. Show
that if A, B G S then ,4 + 5 + i 5 G 5 .
Prove that S is a group under the law of composition
A *B=A +B+AB.
If M is the group of all invertible real 2 x 2 matrices, prove that the groups
S and M are isomorphic.

1.62 Let G = {x G IR \x2 < 1}. Show that if x, y G G then


x +v
1 +xy
Prove that G is a group under the law of composition
x
x *y = #
1+xy

17
Book 3: Groups, rings and fields

By considering the mapping/: G -> IR described by

1 —x'
prove that the group G and the additive group IR are isomorphic.
1.63 Let G be a group and let / : G -+H be a bijection from G to a set #. Prove
that there is one and only one law of compositition * on H such that H is
a group with/an isomorphism.

18
2: Rings and fields

In this section we shall deal with algebraic structures that have two laws of
composition. A ring R is a non-empty set with two laws of composition,
written as addition and multiplication, such that
(1) R is an abelian group under addition;
(2) the multiplication is associative;
(3) these laws are linked by the distributive laws
(\/x,y, z GR) (x + y)z = xz + yz and z(x +j>) = zx + zy.
When R has a multiplicative identity we often say that R is a ring with
a 1, or a ring with an identity. When the multiplication is commutative we
say that R is a commutative ring. The additive identity element 0 is called
the zero of the ring. In a ring it is possible to have xy = 0 with x =£ 0 and
y ¥= 0. Such elements are called zero divisors. A commutative ring with
a 1 which has no zero divisors is called an integral domain. If the ring R has
a 1 and x GR is such that there exists y GR with xy = 1 —yx then we say
that y is an (hence the) inverse of x in R. If every non-zero element of the
commutative ring R with a 1 has an inverse then we say that R is a field.
Thus, R is a field if
(a) R is an additive abelian group;
(b) R \ {0} is a multiplicative abelian group;
(c) the distributive laws hold.
The non-commutative ring Matwxw(IR) of n xn matrices over IR has zero
divisors (for n > 1). The number systems (Q, IR, (D form fields. The number
system Z is an integral domain that is not a field. The ring Z w of integers
modulo n is an integral domain (in fact a field) if and only if n is prime.
As in the case of a group, the notions of substructure and structure-
preserving mapping are important. A non-empty subset S of a ring R is

19
Book 3: Groups, rings and fields

closed (or stable) under the laws oiR if


x,yES=>x+yGS and xyGS,
and is a subring of R if it is a ring under the laws of composition thus inherited
from R. To show that S is a subring it suffices to prove that
x,yeS=>x—yGS and xyGS.
If F is a field then a subfield of F is a subring of the ring F which is also
a field. Note that a field can contain subrings that are not fields; e.g. IR
contains the subfield (0 and the subring Z.
A left (respectively right) ideal of a ring R is a subring / such that
(VrGR)rICI (respectively (\/rGR)IrCI) where rl={ri\iei}. By an
ideal of R we mean a subring that is both a left ideal and a right ideal. Ideals
are to rings what normal subgroups are to groups. Thus for example we can
form the quotient ring R/I where / is an ideal. This consists of the quotient
group R/I under addition, together with the multiplication

For a subset E of a ring R we denote by (E) the ideal generated by E, i.e.


the smallest ideal of R that contains E.
lfR,S are rings then a mapping/: R -> S is a ring morphism if
(\/x,yGR) f(x+y)=f(x)+f(y) and f(?cy)=f(pc)f(y).
A bijective ring morphism is called a ring isomorphism. As with group mor-
phisms there are the important subsets Ker / = {x ER \f(x) = 0} and
Im / = {f(x) | xGR}; the former is an ideal of R and the latter is a subring
of S. The first isomorphism theorem for rings states that for a ring morphism
f:R->Swe have a ring isomorphism I m / ^ R/Kerf.
We shall also consider the ring R[X] of polynomials with coefficients in
a given ring R. In the case where R is a field F, we assume that the reader is
acquainted with the division algorithm and the euclidean division of one
polynomial by another (a(X) = b(X)q(X) + r(X) where r = 0 or
deg r < deg b). Likewise, we assume that the reader can compute the highest
common factor of two given polynomials. An irreducible polynomial is one
that has no proper polynomial factors. For an irreducible polynomial
f(X) = ao + axX + a2X2 + • • • + an_xXn-x + Xn G F[X]
the quotient ringF[X]/(/(X)) is a field which can be regarded as
{b0 + bxx + b2x2 4- • • • 4- bn^x"-1 \ bt G F}
where x satisfies

20
2: Rings and fields
n x
~ axx — a0.
2
For example, the polynomial X 4-1 is irreducible over IR and so IR[X]/
(X2 + 1) is a field that is isomorphic to the field
{bo + btflbtGlR}
where x = - 1 , i.e. IR [X]/(X2 + 1) ss <C.
2

2.1 (0) Let S be the set of mappings / : IR -> IR. Define 4- and • on S by the
prescriptions
(WG IR) (f+g)(a)=f(a)+g(a), (f-g)(a)=f(a)g(a).
Prove that S becomes a commutative ring with an identity. Is S a ring when
multiplication is composition of mappings?
(b) Let S = IR x IR and define + and • on S by
(a, b) + (c9 d) = (a + c,b + d)

Prove that S is a commutative ring with an identity. Find which elements of


S have a multiplicative inverse.
2.2 (a) Let X be a fixed real number. Prove that the set R(X) of matrices
of the form
\x+y J>1
L \y xY
where x, y G IR, is a subring of the ring M= Mat2X2(IR). Show that R(X)
has an identity for every X. Show also that every non-zero element of the
subring R(— 1) has a multiplicative inverse. Is the same true of the subring
R(0)l
(b) Prove that the set F of matrices of the form

where x G IR,x\
1-
is a subring of M. Show that F has an identity element that is
different from the identity element of M. Prove that every non-zero element
of F has a multiplicative inverse. How is this possible when the matrices in
F all have zero determinant?
2.3 Prove that in the ring Z w the set of non-invertible elements consists of zero
and the zero divisors.

21
Book 3: Groups, rings and fields

2.4 Show that the set K of matrices of the form

b a \
where a, & G Z 3 forms a subring of Mat 2X 2(^3)- Prove that K is a field.
Show that the non-zero elements of K form a cyclic group of order 8 under
multiplication. Find a generator of this cyclic group.
2.5 Determine how many solutions the equation x(x + 1) = 0 has in each of the
following rings:
(tf)Z, (Z>)Z6, (c)Z 4 1 , (rf)IR.
2.6 In the ring Z 3 4 find
(a) an element a such that every element of Z34 with a multi-
plicative inverse is a power of a;
(b) all elements x with x 2 = JC.
2.7 Let i? be a cummutative ring. Define binary operations 0 and ® on
E = R x R by

0 , Z>) ® (c, d) = (flc +fed,Z


Show that E is a ring with zero divisors. Prove that if R is an integral domain
then the zero divisors in E are of the form (a, a) and (a, —a).
2.8 If D is an integral domain prove that the equation x2 = 1 has only two
solutions in D. Use this fact to prove Wilson's theorem that if p is prime
then(p — 1)! =—l(modp).
2.9 Let E be a ring with identity e. Define an operation * on E by setting
(Va, 6 G # ) a*b=a + b-ab.
Let G be the subset of # consisting of those elements a for which there
exists an element w in £ with

Prove that G is closed under * and that it forms a group under this opera-
tion. Show also that if aGE then a belongs to G if and only if e — a is
invertible in E.
2.10 Show that the set H of matrices of the form
f a + bi c + dH
l-c + di a-bi\

22
2: Rings and fields

where a, b, c, dG IR is a subring of Mat2X2(^)- Prove that every non-zero


element of H has an inverse. Deduce that H contains no zero divisors. Is
# a field?
Let K be the subring ofH consisting of those matrices of the form
r a + bi c + dil
L—c + di a — b\\
where a,b,c,dez. Find allX GK with X2 = - / 2 .
2.11 Consider the subring Z[i] = {a + \b \ a, b G Z} of (D. For tf + i&GZ[i]
define /(a 4- \b) = a2 + b2. Show that if a, 0 G Z[i] then /(a0) = /(«)/(/}).
Deduce that a G Z [i] has a multiplicative inverse if and only if / ( a ) = 1.
Hence show that 1, —1, i, —i are the only elements of Z[i] that have multi-
plicative inverses.
If a G Z [i] is non-zero and does not have a multiplicative inverse then
we say that a is irreducible if a = f$y requires ]3 or y to have a multiplicative
inverse. Prove that if / ( a ) is prime then a is irreducible. Give an example of
an irreducible element a. of Z [i] with / ( a ) not prime.
Write 43i —19 as a product of irreducibles.
2.12 Let R be a commutative ring with an identity. A subset S of R is said to be
multiplicatively closed if
(1)
(2)
Let M = RxS and define a relation ~ on Af by

(Note that if/? has no zero divisors then this is simply rxs2 = r2Si.)
Show that ~ is an equivalence relation. If Rs is the set of ~-classes show
that addition and multiplication may be defined onRs by the prescriptions

Show that in this way i? 5 becomes a commutative ring with an identity.


(a) If R = Z and S = Z \ {0} show that # £ s* <D.
(ft) If i? is an integral domain and S = R\ {0} show that Rs is
a field.
(c) If i? = Z 6 (which is not an integral domain) and S= {2, 4}
show that the order of Rs is 3.

23
Book 3: Groups, rings and fields

2.13 If p, q G (D prove that p2 — pq 4- q2 = 0 if and only if p = q = 0. Use this


fact to prove that if
n oi ro l l
and A=\
LO lJ L-l - 1 J
then, under matrix addition and multiplication, S = {pA + ql | p, q G (Q} is
a field.
2.14 Let F be a subfield of the field IR and let p be a positive integer such that
Vp € # Prove that if a, b GF then a + b\/p = 0 if and only if a = & = 0.
Deduce that if a,b, c,dGF then a + b yjp = c 4- d\Jp if and only if a = c
and & = J.
Prove thatF[\Jp] = {a + b\fp\a,b GF}is a field.
Suppose now that m and n are positive integers such that \Jm £ (Q,
V« £ ® and sjimri) $ (D. Prove that V ^ ^ Q[V^] a n d deduce that the set
S of real numbers of the form
a + by/m + cy/n + d\/(mn) (a,b,c,dE<&)
is a field.
2.15 Let R be the set of differentiable functions / : IR-HR. Prove that R is
a commutative ring with identity when addition and multiplication are
defined by the prescriptions
(/+ g)(x) = f(x) + *(*), (#)(*) = f(x)g(x).
lfS= {fGR | / ( 0 ) = 0} prove that S is an ideal ofR. If T= {fGR \ D/(0) = 0},
where D/ is the derivative of/, prove that T is a subring of i? which is not
an ideal of R, but that S n 71 is an ideal of i?.
2.16 Let X be a subset of a ring R. Define
A(X) = {rER | (VJC GX)rx = 0};
B(X) = {r G R | (VJC G X)xr = 0}.
Prove that ^4(X) is a left ideal of R and that i?(X) is a right ideal of i?. If
X is a left ideal, prove that A(X) is a two-sided ideal.
Now let R = Mat2 x2 (IR) and let X be the subset of matrices of the form

Prove that X is a left ideal of R and find A(X) and fl(X). Is 5(X) a two-
sided ideal of/??

24
2: Rings and fields

2.17 Let R be a commutative ring and let / be an ideal of R. Define \/l to be the
set of those xGR such that xn El for some positive integer n. Prove that
yjl is an ideal of R, that / C y/l, and that y/l = \/(V0-
If/ is also an ideal ofR, prove that

(b)
2.18 Show that the set R of matrices of the form
r a by/51
l-by/5 aJ
where a, & G Z forms a subring of the ring Mat2X2(|R)- Show that R is
commutative and has an identity.
Show that the subset / of matrices of the form
r x (3y 4- x) %/5]
L—(3y + jc)V5 x J
is an ideal of R. Check that each of the matrices

L-V5 1J LO 3 .
belongs to /. By considering the determinants of these matrices, show that
there is no r G R with I=(r).
2.19 Given subsets A and B of a ring R, define

Which of the following are true? Give proofs for those that are true and
counter-examples (using, say, 2 x 2 matrices) to illustrate those that are false.
(a) If A and B are sub rings then so is A -f Z?,
(b) If A and i? are left ideals then so is A + B.
(c) If A is a left ideal and B is a right ideal then A + 5 is a subring.
(J) If ^4 is a two-sided ideal and 5 is a left ideal then A + B is
a left ideal.
2.20 Let >1 be an additive abelian group. Let Hom(A) be the set of group morphisms
from A to itself. Define an addition on Hom(/4) by (f,g)^f + g where
(WxGA) (f+g)(x)=f(x)+g(x),
and let multiplication be composition of mappings. Show that Hom(/1) is
a ring.

25
Book 3: Groups, rings and fields

If A is the group Z show that Hom(^4) is isomorphic to the ring Z.


If A is the group Z n show that Hom(/4) is isomorphic to the ring Z w .
If A is the group Z x Z show, by considering for example the mappings
described by (JC, y)-+(x9 0) and (x, y)-+(x, x), that Hom(^4) is not
commutative.
2.21 Let R and S be non-trivial rings and f.R^-S a surjective ring morphism.
Determine (giving proofs or counter-examples) which of the following state-
ments are true.
(a) IfR is commutative then so is S.
(b) ifR has an identity then so does S.
(c) IfR and S have identities t h e n / ( l # ) = 1$.
id) IfR has zero divisors then so does S.
(e) IfR is an integral domain then so is S.
( / ) If R is a field then so is S.
2.22 Let F b e the ring in question 2.2(6). Prove t h a t / : IR -> Mat 2X 20R) given by

[2X 2Xj

is an injective ring morphism. Deduce that Fs IR.


Define^ : Mat 2x2 (IR)-> IR by

Is g a ring morphism? Are fog and g o /ring morphisms?


2.23 Let i? be the set of rationals a/b (in lowest terms) such that b is not a multiple
of 3. Prove that R is a subring of (D.
Let / be the subset of i? consisting of those elements whose numerators
are divisible by 3. Prove that / is an ideal of R.
Prove that R/I is a field.
2.24 A ring R is said to be boolean if a2 =a for every flGi?. Prove that if R is
boolean then 2a = 0 for every aGR. Deduce that a boolean ring is
commutative.
Let P(iS) be the power set of a non-empty set S. Prove that F(S) is a ring
under the operations A and n. Show that this ring is boolean.
If ECS prove that ?(E) is an ideal of P(5). If ^ : P(S) -* ?(S) is given by
/ £ ( I ) = I f i £ ' ' (where E' denotes the complement of E in S) prove that
fE is a ring morphism. Deduce that ?(S)/P(E) ^ !

26
2: Rings and fields

2.25 Consider the mapping a : IR [X] -» Mat2X2(IR) given by

i
D/(O) /(O)J
where D/ denotes the derivative of fG IR [X]. Prove that a. is a ring morphism
and determine its image and kernel. Find a polynomial that generates the
kernel of a.
2.26 Prove that R = {a + b\Jl \a, Z> G Z} and S={a + by/3\a, bGZ] are
subrings of (D. Show that R and S are not isomorphic as rings.
2.27 Prove that the natural ring morphism # : Z -» Z 4 given by #(n) = «(mod 4)
maps all odd squares to the same element.
Deduce that no member of the sequence of integers
11, 111, 1111, 11111, ...
is a square.
2.28 Prove that if p is prime then there are exactly two non-isomorphic rings with
p elements.
2.29 Determine (giving a proof or a counter-example) which of the following
statements are true.
(a) Every subring of a field is an integral domain.
(b) Every quotient ring of an integral domain is an integral domain.
(c) The invertible elements in an integral domain form a cyclic
group under multiplication.
(d) If R is a ring with an identity and the non-zero elements of
R form a cyclic group under multiplication then R is a field.
(e) Given a field F there is an integral domain R with F a subring
of/2.
2.30 An element x of a ring R is said to be nilpotent if xn = 0 for some positive
integer n.
(a) If R is a commutative ring show that the nilpotent elements of R form
an ideal N.
(b) Show also that the ring R/N has no non-zero nilpotent elements.
(c) ifR is the ring Z w , what is Nl
id) If R is not commutative show (by considering for example
R = Mat2 X2O R)) that the nilpotent elements do not necessarily form an ideal.
2.31 Let R be a commutative ring with an identity and let ah bj, xGR where

27
Book 3: Groups, rings and fields

i = 0 , . . . , n and/ = 0 , . . . , m. Establish the identity


n m n~hm / k
k \
+/
z z «V = z z «;-/6;)*
where a\ = 0,- for 0 < fc/ < n and
/= O ;=O = O \/ = O /
at = 0 otherwise, and similarly bj = bj for
0 < / < m andfr;-= 0 otherwise.
Hence deduce that, for every x E R, the substitution map £x : /? [X] -> jR
defined by

is a ring morphism.
Given xG IR let f^ : IR[JT]-> IR be the substitution morphism. Deter-
mine the image and kernel of $x. Find a polynomial that generates Ker $x.
2.32 Let &m : Z -> Z m be the 'natural' morphism, described by
#m(fl) = the remainder in the euclidean division of a by m.
Prove that &m : Z[X] -> Z m [X] described by

w=o
is a surjective ring morphism. What is the kernel of # m ?
Find the quotient and remainder on dividing
(a) X 4 + 3X3 + 2X2 + X + 4 by 3X2 + 2Xin the ring Z 5 [X];
(6) X10 by X2 + 1 in the ring Z 2 [X].
Find all positive integers n such that X2 +2 divides Xs — 1 OX + 12 in the
ringZjX].
2.33 (a) Find a commutative ring R with an identity for whichR[X] has zero
divisors. If f(X)ER[X] has constant term 1, can/(X) be a zero divisor?
Can you generalise this result?
(b) Let R be a commutative ring with an identity. Show that 1 + aX is
invertible in R [X] if and only if there exists n > 0 with an = 0. =*
(c) Let F be a field. Find all the invertible elements of F[X],
2.34 If R is an integral domain prove that so also is R[X]. If R is a field, prove
that R [X] is never a field.
Prove that any polynomial in Z 4 [X] which has a multiplicative inverse
has constant term either 1 or 3. If

28
2: Rings and fields

p(X) = a0 + ^X+ • - - + anXn


satisfies a0 = 1 or 3 and at = 0 or 2 for / > 1, prove that p(X) is invertible.
2.35 Show by means of counter-examples that the division algorithm fails in

2.36 Find the highest common factor of each of the following pairs of polynomials
and express it in the form a{X)f(X) + b(X)g(X):
(a) /(X) = X 3 - 1 , s(X) = X 4 - X 3 + X 2 4 - X - 2 in the ring

(b) f(X) = X2 + l,g(X) =X 3 + 2X- i in the ring (D[X];


(c) f(X) =X 4 + 1,#(X) = X2 + X + 2 in the ring Z 3 [X].
2.37 Consider the set F of matrices of the form al2 + bK where I2 is the 2 x 2
identity matrix and

-1
Prove that F i s a subfield of the ringMat2X2(IR).
Show that the polynomial X2 — X 4-1 is irreducible over (Q and prove
that the fields (D[X]/(X2 - X + 1) and F are isomorphic.
2.38 Show that the set R of real matrices of the form
a b c
O d e
0 0 /
forms a subring of the ring Mat 3x3 (IR).

a the
Show that b cmapping a : / ? - H R x l R x l R given by
0 d e
0 0 / >(a,d,f)

is a ring morphism. Find an ideal / of R with R/I s* Im a.


2.39 Compute "the
3 product -r
-5 I
Vc dl Ll 1
. 1 I.

29
Book 3: Groups, rings and fields

Deduce that & : Mat 2x2 (IR) -^Mat3X3(IR) given by

•L A-
6a + 3b-2c-d 3a + 3b-c-d 3a + 6b-c-2d
—10a — 5b + 2c + d —5a — 5b + c + d —5a — 10b 4- c 4- 2d

is a ring morphism. Show that I m i } ^ Mat 2x2 (IR).


2.40 Find the highest common factor h(X) in the ring (Q[X] of
f{X) = X 5 + X 4 4- 2X3 + 2X2 + 2X4-1 and

Show that the smallest ideal / of (D[X] which contains / ( X ) and #(X) is
{k(X)h(X)\k(X)e<b[X]}.
Prove that (Q[X]/7 is isomorphic to the subring S — {a + b \/3 i | a, b G (D}
of(D.
2.41 Consider the polynomials

Prove that in Z 5 [X] the highest common factor of /(X), ^(X) is X2 + 2X+ 2
whereas in Z 7 [X] it is X + 4.
2.42 Given q £ (D let ^ be the set of real matrices of the form al2 + bK where
0, Z> G (D and

=
l i oJ-
Show that Sq is a subring of Mat2X2(®) t n a t i s commutative and has an
identity. Prove that Sq is a field if and only if \/q $ (D. In this case show that

2.43 Let i? be a ring with an identity and let / be an ideal of R. If / contains an


invertible element of R, prove that / = R.
Suppose now that R is the ring of upper triangular real 2 x 2 matrices,
i.e. matrices of the form

Lo z\
30
2: Rings and fields

where x,y,z E IR. Prove that

H[o ol'H
are ideals of R and that, apart from R and {0}, these are the only ideals of JR.
2.44 Let /(X) E Z 2 [X] be irreducible with deg f(X) > 2. Prove that f(X) has an
odd number of non-zero coefficients.
Find all the irreducible polynomials in Z 2 [X] of degree less than or equal
to 4. Hence give examples of fields with 4, 8 and 16 elements.
Use the above results to write the following polynomials in Z 2 [X] as
products of irreducible polynomials:
(a) X 5 + X 4 + l;
(b) X 6 + X 5 + X 2 + X + l ;
(c) X

0) X
( / ) X 8 + X 6 + X 5 + X 4 + X3 4- X 2 + 1.
Show that Z 2 [X]/(X4 + X + 1) is a field in which the multiplicative group
of non-zero elements is cyclic and generated by the image of X under the
natural morphism from Z 2 [X] to Z 2 [X]/(X4 + X + 1).
Show that the image of X in Z 2 [X]/(X4 4- X 3 + X2 + X + 1) does not
generate the multiplicative group of non-zero elements but, by finding
a generator, show that this group is cyclic.

31
Solutions to Chapter 1

1.1 If [a]= [x] and [b]= [y] then a—x — np and b—y=nq for some p,
Then
ab—xy — ab — xb + xb— xy
= (a ~x)b + *(Z> -y) = n(pb + <pt)
and so [#Z>] = [jey]. Similarly
a + Z> — (x 4-;;) = «p 4- nq — n(p + q)
gives [0 + Z>] = [JC + y].
[2][x] = [3] has no solution.
[5][x] = [0] has precisely one solution, namely [0].
[2][x] = [2] has two solutions, namely [1] and [4].
1.2 Observe that n2 = (n + 6fc)2(mod 6) for all integers n and k, so that nR(n + 6k).
Thus we need inspect only the equivalence classes of the integers 0,1,2,3,4,5.
Now it is readily seen that, modulo 6, we have
0 2 = 0 , 12 = 1 = 5 2 , 3 2 = 3 , 4 2 = 4 = 2 2 .
There are therefore only four R-classes, namely

That [x]R [y]R = [xy]R defines a binary operation follows from the observa-
tion that if aRx and bRy then a2 = x2 + 6p and b2 = y2 + 6q whence a2b2 =
(x2 + 6p)(y2 + 6q) = x2y2 + 6r where r G Z , and so abRxy. The corres-
ponding Cayley table is then

32
Solutions to Chapter 1

[0]* [1]* [2]* [3]*

[0]* [0]* [0]« [0]« [0]*


[l]tf [0]* M«
[2]* [0]* [2]* [2]* [0]*
[3]« [0]* [3]* [3]«
[0]R
That [X]JJ + [y]n = [x + j>]# does not define a binary operation on S follows
from the observation that if aRx and bRy then a2 = x2 + 6p and 6 2 =y2 + 6#
and so
(a + Z>)2 = (x+j;) 2 - 2xy 4- 2a6 + 6(p + ^r)
from which we see that (a + b)R(x +y) if and only if ab =xy(mod 3). This
is not always the case; take, for example a = b=x=l,y = —l.
1.3 The Cayley table for * is readily computed and is the following:
* 2 3 5 7 11 13
2 2 3 5 7 11 13
3 3 2 3 2 3 7
5 5 3 2 5 7 2
7 7 2 5 3 2 3
11 11 3 7 2 5 11
13 13 7 2 3 11 3
It is clear from this table that 2 is an identity element. Moreover, every
element has an inverse:
2 has inverse 2;
3 has inverses 3 and 7;
5 has inverses 5 and 13;
7 has inverses 3 and 11;
11 has inverse 7;
13 has inverse 5.
Finally, * is not associative since with respect to an associative law no
element can have more than one inverse.
1.4 That the operation * is associative follows from a purely routine verification
of the associative law. In searching for an identity element we look for an
element (V, / , z) such that (JC, y9 z) * (V, y\ z') = (x, y, z) for all x,y9 z\

33
Book 3: Groups, rings and fields

i.e. we have to solve


xx = x\ ay' + yx' —y\ otz' + zx' = z.
Clearly, x = 1 and y = z = 0 satisfy these requirements. Consequently
(1, 0, 0) is the identity element with respect to *.
Now (x, y, z) has an inverse (x\y',zr) if and only if (x,y,z) *(x',y' ,z') =
(1, 0, 0); so given (x, y, z) we have to solve the equations
xx=\\ ay'+yx' = 0; az' + zx = 0.
These equations can be written in matrix form
X 0 0" V "1"
y a 0 / = 0
_z 0 a. .0.
and we know from matrix theory that a unique solution exists if and only if
the coefficient matrix A is such that det Ai=0. Now clearly det A =XOL2.
Hence a solution exists if and only if x and a are non-zero, in which case the
solution is given by
r 1 t y t z
X —X — y = — —, z = — —.
' ax: ax:
1.5 (a) Associative; identity is 0; inverse of In is — 2n; a group.
(b) (1 — 1) — 1 =£ 1—(1 — 1) so not associative;no identity;not a group.
(c) Associative; identity is 1; 0 has no inverse; not a group.
r s
(d) Associative; identity is 1; inverse of - is - ; a group.
s r
(e) Associative operation on Z 6 but Z 6 \ {[0]} is not closed under
the operation; for example, [2] [3] = [0].
( / ) Associative; identity is [1]; [0] has no inverse; not a group.
(g) Associative; identity is 1; inverse of e 2wrl / 3 is e ~ 2mr1 / 3 ; a group.
1.6 (a) No : 1 is tne identity but 2 * a = 1 implies max{2, a} = 1
which is impossible.
(b) No : there is no identity element (if there were such an element
e then from a * e = a we would have min {a, e} = a whence
a < e for all a G Z and e would be the greatest element of Z).
(c) No : 0 is the identity but 1 has no inverse since 1 * b = 0 implies
1 4- b — b = 0 which is nonsense.

34
Solutions to Chapter 1

(d) No : the operation is not associative; for example, we have


2 * (3 * 4) = 2 * 1 = 1 and (2 * 3) * 4 = 1 * 4 = 3.
(e) Yes : (0, 0) is the identity; (—0, —b) is the inverse of (#, b).
( / ) No: the operation is not associative; for example we have
((1, 2)* (2, 3))* (2, 1) = (25, 17) and (1, 2) * ((2, 3) * (2, 1)) =
(17,15).
(g) Yes: (1, 0) is the identity and the inverse of (a, b) is
l_l *_\
2
W+b ' a +b2)'
2

(Think about the non-zero complex numbers under multi-


plication!)

1.7 It is clear that + is a law of composition on T which is associative since


addition of matrices is associative. The identity element of Tis the 2 x 2 zero

[ x yl

—y x i
is
\—x —y~\

Ly —XJ
. Thus T is a group

under+. Now
F* yUx' / ] r xx''-yy' x
Y—y x\ LL—
y x\ —yy {.—(xy'+yx1)
x\ {.—(xy'
x\ xx—yy
so multiplication is a law of composition on T which is associative since
multiplication of matrices is associative. Clearly, the 2 x 2 identity matrix

is the identity element of T. Now det = x2 + y2 and this is 0 if


Y-y x\
and only if x = y = 0. Thus the set of invertible elements of T under multi-
plication is T\ {0}, which is then a group.
1.8 Suppose that / : JC -> ax + b(a =£ 0) and that g : x -> a*x + Z>*0* # 0). Then
/ [g(x)] = f(a*x 4- b*) = a(a*x + Z?*) + b = aa*x + ab* + b
where aa* =£0. Thus fOgGG and so the mapping (f9g) ^fog describes
a law of composition on G which is clearly associative. Also it is evident that
G has an identity element under this law, namely the identity map idiR on IR.
With / and g as above, we have that fog = id)R =g of if and only if,
forallxGIR,
a*(ax + b) + b*=x= a{a*x 4- b*) + b,
i.e. if and only if aa* = 1 and a*b + 5* = 0 = #&* + &. This is the case if

35
Book 3: Groups, rings and fields

and only if
1 b
a* —- an( j £* = —
a a
1
Since a =£ 0 implies - =£ 0, we have that

1 5
# : JC->-# — -
a a
is the inverse of/in G. Hence G is a group.
1.9 The operation * is associative. To see this, we must check several cases. For
example, suppose that a < 0 and b < 0; then we have
a ac b ac
(a * b)* c=- *c = — and a*(b*c) = a*-= —.
b b c b
Similar arguments in the cases a < 0, Z? > 0 and 0 > 0, b < 0 and a > 0,
Z? > 0 complete the proof of associativity.
1 is the identity element. In fact 1 * # = 1# = #; and for a > 0 we have
a * l = al= a, while f o r a < 0 w e h a v e
a
a * 1 = - = am
1
1
If a > 0 then - is its inverse; for we have
a
1 1 1 1
a * - = a - = 1, - * 0 = - 0 = l.
a a a a
If 0 < 0 then # is its own inverse; for we have
a
a * # = - = 1.
a
Hence the operation * makes IR \ {0} into a group.
1.10 See Fig. SI.1.
(a) The only symmetries are the identity and the reflection in the broken
line shown. Hence the group of symmetries is the same as the symmetric
groups.
(b) The group of symmetries is the same as that of the non-square
rectangle, as is shown by the diagram. Thus it is the Klein 4-group V4.

36
Solutions to Chapter 1

Fig.Sl.l

(c) The group of symmetries is the same as that of the square, as is


shown by the diagram. Thus it is the dihedral group Z)4.
(d) The only symmetries of the swastika are the rotations through
multiples of TT/2. Thus it is clear that the group of symmetries is C4.
1.11 Given an w-sided regular polygon, label a pair of adjacent vertices A and
B. In performing a symmetry there are n vertices to which we can move
A; and once the position of A has been fixed there are two (adjacent) vertices
from which to choose the position of B. This then determines the position
of the entire polygon. Thus we see that in all there are 2n symmetries.
These symmetries consist of
(a) rotations through (2ir/n)k where k = 0, 1 , . . . , n — 1;
(b) reflections in lines joining the centre of the polygon to its
vertices, or to the mid-points of its edges. (These are the same
when n is odd.)
Since (a) and (b) account for n symmetries each, they describe all of them.
To see that the group is non-abelian let A and B be adjacent vertices.
Consider the rigid motions /, g described respectively by a reflection in the
line joining A to the centre of the polygon, and a rotation through 2irln.
Then/Og andg o/have different actions on B. Draw pictures to see this!
1.12 Consider an adjacent pair of symbols. If a symmetry leaves this pair in the
same order then clearly it is of the form an where n is the number of units
in the translation (negative values of n represent translations to the left). On
the other hand, if a symmetry reverses the order of the pair then clearly it
is of the form ban. Now to be able to multiply two symmetries of the form
bman and obtain another symmetry of this form, it suffices that ab should
be of this form; and it is easy to see that indeed ab = ba'1. Hence the group
of symmetries is generated by a and b subject to the relations b2 = 1,
ab = ba'K
There are several ways of exhibiting the seven different possible patterns
(either with symbols, letters, pictures, etc.). Here is one way:

37
Book 3: Groups, rings and fields

(1) The symmetries of the pattern


• • -ccccc • • •
consist of translations only.
(2) The symmetries of the pattern
• • • / / / / / • • •

consist of translations and rotations.


(3) The symmetries of the pattern
. . . 33333 •••
consist of translations and horizontal reflections.
(4) The symmetries of the pattern
• • •VVVVV•• •
consist of translations and vertical reflections.
(5) The symmetries of the pattern
•••xxxxx-• •
consist of translations, rotations, and both horizontal and vertical reflections.
(6) The symmetries of the pattern
• • •ununu • • •
consist of translations, vertical reflections, and 'slide reflections' (i.e. a trans-
lation followed by a horizontal reflection).
(7) The symmetries of the pattern
...3^3...
consist of translations and 'slide reflections'.
1.13 A rotation is fixed when we describe what happens to two adjacent corners.
We can put the first corner into one of eight different positions and then
rotate the other into one of three adjacent positions. Thus we see that
\G | = 8 x 3 = 24.
Consider now rotations x that fix a particular corner. They all satisfy
x 3 = 1. There are eight corners, but a rotation that fixes one corner also
fixes the one diagonally opposite. Thus we can find eight different non-
identity elements of this kind (two for each pair of opposite corners).
Similarly, rotations of 90° about the centre of a face satisfy x 4 = 1. So
do rotations of 180° and 270°, and it should be noted that the rotations
of 180° satisfy x2 = 1 also. Such rotations send opposite pairs of faces into
themselves. So, when we count the non-identity ones, we find that there are
nine (namely, three for each pair of opposite faces).

38
Solutions to Chapter 1

Similarly, rotations of 180° about the mid-point of an edge satisfy x2 = 1,


and there are six of these (namely, one for each pair of opposite edges).
Collecting these together, and remembering the identity, we have accounted
for 1 4- 8 + 9 + 6 = 24 elements of G, i.e. all of G.
It is an easy matter to do the same for the octohedron. Simply note that
the centres of the faces of a cube lie at the corners of an octohedron (draw
a picture!), so a rotation of the cube gives a unique rotation of its inscribed
octohedron, and vice versa.
(Remark. The octohedron is called the 'dual' of the cube. The dual of the
icosohedron is the dodecahedron. You may wish to describe in a similar way
the elements of its rotation group. In this case we have | G \ = 60.)
1.14 We use the fact (which we shall refer to as (*)) that every element of the
group appears once and once only in each row and column in the Cayley
table. It is immediate from this that ay — z and az = x, which completes the
second row. As for the third row, we observe that a2 = b and so a3 =ab = ba.
But it is given that ab — e; hence ba = e. Also, bb=a2a2 =a4 =a3a = ea = a.
We can now use (*) to complete the third row as well as the second and
third columns. Consider now the element xx. We have clearly xx G{a,b, e}.
Since xz = a we cannot have xx = a; and if xx = b we have the contradiction
xb = x3 = bx. Hence we must have xx = e. It follows that xy = xxb = eb = b
andyx = axx = ae = a. The rest follow by (*). The complete table is
e a b X y z
e e a b X y z
a a b e y z X

b b e a z X y
X X z y e b a
y y X z a e b
z z y X b a e
1.15 If k is the order of gG G then g generates a cyclic subgroup of order k and
hence, by Lagrange's theorem, k\n. Also,g"fc = 1 and hence g" = 1.
If an integer a = 0(mod p) then it is self-evident that aP = tf(mod p). If
0^O(mod p), calculate in the group Z p \ { 0 } . This is a group of order
p — 1 and hence ap~l = 1 by the above. Consequently, aP = a.
(a) Since 11 is prime, we have 7 10 = l(mod 11) and so 71OO = 11O = 1
(mod 11).

39
Book 3: Groups, rings and fields

(b) Since 13 is prime, we have 912 = l(mod 13) and so 936 = 1 3 = 1


(mod 13), whence 9 37 = 9(mod 13).
1.16 (a) From the information given, the result is true for n = 1. This then
anchors the induction. As for the inductive step, we have (supposing that
the result holds for n)
an+1b = aanb = abakn = bakakn = bak(ri+1\
(b) Again, from the information given, the result is true for n = 1. The
inductive step in this case is as follows:
abnb = bnaknb
= bnbakkH by (a)

That (ban)2 commutes with a can be seen, for example, from the equations
a(ban)2 = abanban = abbankan = nk n+1
a +

(banfa = banbana = bbaknana = a


kn+n+1

and that it also commutes with b can be seen from


b{banf = bbanban = anban = baknan = ba^k+1)n
(ban)2b = banbanb = banbbakn = ba^k+1>>n.
1.17 a = l or b = l ox a = b contradicts ab + ba\ so l,a, b, are distinct. Consider
now ab; we cannot have ab = 1 (since then b=a~l which contradicts abi=ba)9
nor can we have ab =a (for then, by cancellation, b =1). Similarly, ab i=- b.
Thus l,a,b,ab,ba are distinct.
Now a2 ¥^a (otherwise a = 1 by cancellation); a2 ^b (otherwise ab = ba);
a ¥=ab (otherwise a = b by cancellation); and a2 ¥= ba (otherwise a = b by
2

cancellation). Thus we see that either a2 = 1 or a2 $ {1, a, b, ab, ba}. Note


that the latter implies that G contains at least six elements. Suppose then that
the former holds and consider aba. We have that aba ^ 1 (otherwise ab =
a'1 = bd)\ abai^a (otherwise ab = 1); aba^b (otherwise, since a2 — 1 by
hypothesis, ba = a2ba = ab); abai^ab (otherwise a = 1 by cancellation);
and aba ¥" ba (otherwise a = 1 by cancellation). Thus in this case G also
contains at least six elements.
The Cay ley table for F under o is precisely that given in question 1.14,
with a, b, x, y, z replaced by p, q, a, c, b respectively. Thus F is a group
under o.
1.18 (a) Since ab = ba we have b'1 ab = b~lba = a. Multiplying on the right

40
Solutions to Chapter 1

by b~l we obtain b~la = ab~l. Also, since a and b commute, we have


xax~l .xbx'1 =xabx~1
= xbax'1 = xbx'1. xax~\
(b) If G is abelian than (ab)'1 =b~1a~1 =a~lb~l. Conversely, if this
holds for all a,bEG then writing a'1 for a and b'1 for b we obtain ba = ab.
(c) If (ab)2 = a2b2 then we have abab — aabb and so, multiplying on the
left by a'1 and on the right by b~l we obtain ba = ab,
(d) If a2 = 1 for all a G G then given any x, y £ G we have (xy)2 = 1,
x2 = 1 and jy2 = 1. Consequently (pcy)2 = x2.y2 and the result follows by (c).
(e) The result is trivial for n = \. For the inductive step, suppose that
(x~lyx)n = x~lynx\th.Qn
(x~1yx)n+i = (x~1yx)nx~1yx — x~lynxx"lyx = x~1yn+1x.
( / ) We are given that b6 = 1 and ab = b4a. From the second of these and
(e) we obtain b = a~lb*a = (a'lbaf. Thus
b3 = (a-'ba)12 = a~lbl2a = a~l\a = \,
and ab = &4# = b3ba = ba.
ro n
1.19 The orders are respectively 6,3,4. The element has order 2, as does
LI OJ
[-1 01
(and many more!).
The given formula is clearly true for n = 1. As for the inductive step,
, fl kl
suppose that A —\ ; then

LO 1J LO U LO 1
It is clear from this that A does not have finite order. Since the subgroup
generated by A is thus infinite, so also is G. Finally, taking a =A, b =/ 2 and
|~1 01
c=\ , we see that a and b commute, as do b and c, but a and c do

not commute.
[1 1]
1.20 (a) Cyclic : every element is a power of . The group is then

abelian. It is also infinite.

41
Book 3: Groups, rings and fields

(b) Cyclic : every element is a power of [2]. The group is then abelian.
It is finite.
(c) Not abelian : for example, (123) o (12) =£(12) o (123). Thus the
group cannot be cyclic. It is finite.
(d) Cyclic : every element is a power of (12). The group is then abelian.
It is finite.
(e) Cyclic : every element is a multiple of 2. The group is then abelian.
It is infinite.
( / ) Abelian but not cyclic : for if it were cyclic then every real number
would be a multiple of some r E IR, whence 1 would be a multiple of r and
r would be rational; but then, for example, n cannot be a multiple of r.
Clearly infinite.
1.21 If a ¥=• 0 and c^O then ac^O, so the given prescription defines a law of
composition on G. To see that it is associative, check that
[(a, b)(c, d)lf,g) = (acf, bcf+df + g) = (a, b)[(c, d)(f9g)].
The identity element is (1, 0); and it is readily seen that

(a, h)
~\a' a)'
Clearly, the group G is non-abelian.
It is easy to check that H, K, M are subgroups of G. For example, if
(y, 0)GK then (y, 0)" 1 = (y~\ 0) and hence, given (x, 0) and (y, 0) in
K, we have
(pc, 0)(y, Oy1 = (*, 0)(y~\ 0) = for1, 0)GK.
As for L, except for the case n = 0 (when L = K), the subset L does not
contain the identity element (1, 0) and so cannot be a subgroup.
(—1, a) is an element of order 2 for every a G IR; for
( - 1 , a)(~l, a) = (1, -a + a) = (1, 0).
Suppose now that (a, b) has order 3. Then
(l,0) = (a,b)3=(a\b(a2±a + l))
so a ~ 1 and b = 0. Thus (a, b) = (1, 0), which has order 1. We conclude
that G has no elements of order 3.
1.22 (a) If x, y GH then x = 2n, y = 2m for some n, mGZ and so xy'1 =
n m
2 ~ Gi/. Hence H is a subgroup.
If x,y €:K then f o r n , m , r , s E Z w e have

42
Solutions to Chapter 1

l+2n l + 2r
x =
1 + 2m 1 + 2s
That jfiT is a subgroup now follows from the fact that
,
1
l+2/i 1+25 1 + 2 0 + s +
xy " GK.
1 + 2m 1 + 2r 1 + 2(m + r + 2mr)
(6) Check that [a], [b] EH implies [a] - [b] GK
(c) The subgroups are
{e}, {e,x}, {e,y}, {e,z},
{e.a.b}, {e,a,b,x,y,z}.
(Note that all subgroups except the whole group are cyclic.)
1.23 It is a routine matter to show that

(x1x2x3, (xi + yx) (x2 + y2)(x3 + y*) -


whence associativity follows by symmetry. The identity element is readily
seen to be (1,0). Also, every element has an inverse, that of (x, y) being

(,.„)-- (I. - _
Thus 5 is a group.
To verify that P is a subgroup of S, let x = (1, a) GP and y = (1, 6) GP.
Then

and this belongs to P since 1 + a > 0 and 1 + Z? > 0.


G is, by definition, the set of all 2 x 2 matrices whose elements are integers
and whose determinant is 1. It is an easy matter to show that G is a group
under multiplication.
\a bl \ d -bl
The inverse of G G is .To show that HN is a subgroup,
Ic d\ L—c a J
\a b~] \a b'l
let and be in HN; then

Va bl r d' -b'irad'-bc'-ab' + ba'-)


[c d\[-c' a' llcd'-dc' -cb' + da'l

43
Book 3: Groups, rings and fields

Fig.S1.2

and, using the result of question 1.1, it is easily seen from the definition of
HN that this product matrix belongs to HN.
The Hasse diagram for the set {HN | 2 < J /V< 12} is as shown in Fig. SI.2.
1.25 Let G denote the multiplicative group Z p \ {0}.
(a) If #, b GQ then there exist x, y EG such that x2 = a and y2 = b.
Since G is abelian, we have Cry"1)2 = x2(y2)'1 = ab~\ whence ab'1 G Q.
(b) Consider a few examples to see what is happening (for example,
p = 3, 5, 7). You will soon observe that the only time two distinct numbers
have the same square is when one is minus the other (for, in Z p we have
x2 =y2 if and only if (x — y)(x + y) = 0). It follows that only half the
numbers in G give distinct squares, so we can take for Q the set

(c) If nqGQ then nq = x2 for some x. Since q—y2 for some y, we


obtain the contradiction n = (xy'1)2.
(d) Note that for every b EG we can solve the equation mx = b. Now
mq&Q for q G Q (by (c)). If then we had mn$Q for some n $ Q then
there would not be enough elements of the form mx to 'go round' Q. Hence
we must have mn G Q.
(e) By the first part of question 1.15 (which is often referred to as the
corollary to Lagrange's theorem) and (b) above, we have ^(P™ 1 )/ 2 = i
(modp).
1.26 To check that M is a subgroup of the group of all 2 x 2 real invertible
matrices, note that M(0) = / and that M(d)M(if) =M(& + </?). Hence, given
M(d) and M(y) we have

Also, M(d)M(if) = M(& + <p) which clearly implies that M is abelian.


/2TT\
Observe now that Ml — I is an element of order n. In fact

44
Solutions to Chapter 1

MIn l-j\ = M(2ir) = M(0) =,

and if 0 < m < n then


M(2irm\
Kir)] = h
which is not the identity. Thiis \M I — I ) is a cyclic group of order n.

Hence M has cyclic subgroups of every order n.


The answer to the last part is V e s ' -M(l) generates an infinite cyclic
subgroup. To see this, observe that if [M(oc)]n =1 then na = 2-nm for some
integer m so that a is a rational multiple of 2n. Since clearly 1 is not a rational
multiple of 2?r it follows that M(l) is of infinite order.
1.27 If G = (g) is of order n then the subgroups of G are of the form (gt).
Suppose that hcf {r9 n} — d =£ 1. Since gr = gd^r^d^ we have that £ G {gd).
Since d = n/t for some t^n the subgroup (gd) is of order ti^n. Thus
grE(gd)¥z G so {£) is not the whole of G. We conclude from this argument
that the other generators of G are gr where hcf {r9 n} = 1.
By the above, the generators of Zi 8 are 1,5,7,11,13,17. Consider the
subgroups generated by the remaining elements.
<2>= {0,2, 4, 6, 8, 10, 12, 14, 16},
a subgroup of order 9. It has generators of the form r2 where r, 9 are coprime.
Thus r = 1, 2,4, 5, 7, 8 and so r2 = 2, 4, 8, 10, 14, 16.
(6) = { 0 , 6 , 12},
of order 3. Its generators are of the form r6 where r, 3 are coprime. So
r= 1,2 andr6 = 6, 12.

Fig.S1.3

<16> = <14) = < 10) = <8> = <4> = <2>

45
Book 3: Groups, rings and fields

The only subgroups not considered so far are those generated by 3,9,15.
Now
<3>= {0,3,6,9, 12, 15},
of order 6. The generators are r3 where r, 6 are coprime. So r=l, 5 and
r3 = 3, 15. Finally
<9>={0,9}
and the subgroup Hasse diagram is as shown in Fig. SI.3.
1.28 Suppose that x, yGHOK. Then x, yGH gives xy'1 EH and similarly
x, y G K gives xy~ G K. Thus xy~l G H D K and H n £ is a subgroup.
l

Let gGHOK. Then the order of £ divides both | # | and \K\. Since by
hypothesis we have hcf {\H\, \K\} = 1 it follows that the order of g divides
l,whence£=l.Thusi/n#={l}.
Clearly raZ n «Z = kZ. where A: = lcm {m, n}.
1.29 Let x, y G C(a). Then, using the result of question 1.1 S(a), we have
xy'1 .a =x.y~1a = x.ay~1 =xa.y~1 =ax.y~1 =a.xy~1
and so xy'1 G C(a). Hence C(a) is a subgroup of G.
C(H) = naG//C(flf) and so C{H) is a subgroup of G since the intersection
of any family of subgroups is a subgroup.
(a) G = H=C2.
(b) G = H = S3.
(c) G = C4 and H= (a2) where C4 consists of powers of a.
1.30 Note first that abi=l; for ifab = l then we have the contradiction a = b~1 =b.
Now since G is abelian we have (ab)2 — a2b2 = 1.1 = 1.
Let H= {1, a, b, ab}. To show that H is a subgroup of G it is enough to
show that it is stable under multiplication. This is easily seen to be the case
since a2 = 1 = b2 and G is abelian. Now 1 has order 1 and a, b, ab all have
order 2. Since \H\ = 4 it is clear that H cannot be cyclic.
[2*— 1] has order 2; for, k > 1 so
(2k - I) 2 = 22k - 2k+l + 1 = 1 (mod 2k+1).
Similarly (2 fc + I) 2 = l(mod 2k+1) shows that [2^4-1] is of order 2. But
[2k — 1] =£ [2k + 1] forfc> 1 and so, since G is abelian, we have that

is not cyclic. But every subgroup of a cyclic group is cyclic. So if G were


cyclic we would have the contradiction that H is cyclic. We conclude that
G cannot be cyclic.

46
Solutions to Chapter 1

1.31 We observe first that


Pi 1 / 1 \
ri=— =Piq2 ^( >
qi qiq2 *qiq2
and similarly
/ 1 \ / 1 \
r2E{ ) , so that (rl9r2)Q( )

Suppose now that rt = — G (Q for / = 1 , . . . , « . Then we have


qi
1 / 1
. . Qy, ' G <

and so we have that

rur29. ..,rn)c( \ .
\q1q2^ *qn'
Now the group on the right is cyclic (being generated by a single element);
and every subgroup of a cyclic group is cyclic. Hence (rt, r2,..., rn) is cyclic.
Clearly, each Ht is cyclic. We have to show that Ht CHi+1 withH t
Now

Hence //,- C Hj+i; and the inclusion is strict since


1

To show that (D = ^n^\Hn we must show that, given any rational — with
b
b > 0, we have
a
~G//,- for some/.

But this follows from the observation that

- = {b-\)\a-^Hb.
b b\

Finally, suppose that (D were cyclic, say (Q = ( - V Then

47
Book 3: Groups, rings and fields

{ nab
which gives ®. = Hb, contradicting the fact that the chain of subgroups
Hf is infinite.
1.32 Show that (a) => (e) => (d) => (b) =» (c) => ( / ) => (a).
(a) => (e) If xH = Hx then for every hEH we have x/z G:*:// = iftc so that
there exists ti EH with xh = h'x.
(e)=>(d) x-'h = tix~l givesx-'hx = ti EH
(d) => (b) This is obvious.
(b)=*(c) Since x~lHxCH for every xEG we have (writing x" 1 for x)
that ( x " 1 ) " 1 ^ " 1 C/f, i.e. that xHx'1 CH. The reverse inclusion comes from
the fact that xhx'1 = ti EH gives h = x~ltix E x'lHx.

( / ) =* (a) If h~lx~lhx = h'EH then we have hx = xhti. Thus if t EHx


we have t = hx=xhtiExH so that HxCxH. Likewise, h~1(x~1)~1hx'1 =
h*EH gives xh=hh*x and so if f e j t # then t = xh = hh*x EHx. Thus
x/f = i/x as required.
1.33 N is normal in G if and only if, for all* EG and all w EN we have x~1nxEN,
i.e. nx ExN, i.e. nxN — xN.
Take N— 3Z which is a normal subgroup of the additive group Z. Then
1 + 3 + 1 = 5 ^ TV shows that (a) and (&) are not equivalent.
1.34 If # is of index 2 then H has only two right cosets and likewise two left
cosets. Each of the coset decompositions must therefore coincide with
{H, G \ H}. Thus ifx&Hwe have xH=G\H = Hx, whereas if x EH then
clearly xH = H = Hx. Thus we have xH = Hx for every x E G so H is normal.
It is readily seen that products of elements of A U B behave as follows:
A ifX,YEA\
A ifX, YEB;
XYEl
B if XEA,YEB\
B ifXGB, YEA.
Thus A U B is stable under multiplication. Since every element of A U B is
invertible, with XEA^X~lEA and X e t f ^ X ^ G j ? , it follows that
A U B is a group.

48
Solutions to Chapter 1

The above observations on products also show that A is a subgroup of


AUB and that the cosets of A in A U B are A and B. It follows by the first
part of the question that A is a normal subgroup.

The set P of matrices of the form is clearly a subgroup of A, and


o l
is a normal subgroup of A since these matrices commute with every matrix
in A. However, P is not a normal subgroup of A U B; for example, if
\2 01 TO 11
X and Y=\
Lo 1J Ll 0J
then we have
n oi
Y~XXY WP.
Lo 2i
1.35 It is readily checked that G is a group. The identity element is the 3 x 3
"1 a b —a ac—b
identity matrix and the inverse of 0 1 c is the matrix 1 -c
.0 0 1. 0 0 1
If h E H then h commutes with every element of G; for
1 X y 0 a 1 x y +a
0 1 z 0 1 0 0 1
0 0 1. 0 0 1. 0 0
1 0 a 1 X /
0 1 0 0 1 z
0 0 1 . .0 0 1.
Thus gH = Hg for every g E G and so H is normal.
It is easy to check that AT is a subgroup of G. Now K is not normal in
1 0 o"
G. For if x = 0 1 1 then xK is the set of matrices of the form
.0 1 1.
1 a 0 "l a a
0 1 1 whereas Kx is the set of matrices of the form 0 1 1 , so
.0 0 1. .0 0 1

49
Book 3: Groups, rings and fields

1.36 The property that there exists a positive integerrcsuch that (\/x,yE:G)(xy)n =
xnyn may be expressed in the form : the mapping f \G->G described by
f(x) — xn is a morphism. Consequently Gn = Im / is a subgroup of G which
is normal since gxng~l = (gxg~l)n £ Gn for every g G G. Also, Gw = Ker / is
a normal subgroup of G. Applying the first isomorphism theorem we see that
Gn = I m / s G/Ker/= G/Gn.
1.37 (a) (1478)(265)(39).
(b) (18)(364)(57), (134)(26)(587), (13478652).
(c) (13745).
1.38 Yes : (12)(34).(13)(24) = (14)(23) = (13)(24).(12)(34);
No : (12)(24).(13)(34) = (13)(24) but (13)(34).(12)(24) = (12)(34).
1.39 Consider a cycle o = (i1i2 . . . zr). Clearly we have c/+1 = a and so ar=\.
Also, for s < r we have os+1 =£ a and so 0s ¥" 1. Hence a is of order r.
Suppose now that o = a$ where a, |8 are disjoint cycles of order r, s respec-
tively. Let t = lcm {r, s}. Since disjoint cycles commute we have or* = o:^.
Now r \t and 51 ^, and (/ = 1 = jS5; hence o;^ = 1 = $ and consequently crf = 1.
But t is the least integer with this property, since for u < t we have aM/3M = 1
implies a" = 1 = /3". It follows that a has order t.
a = (138)(27)(4965).
The order of o-is lcm {3,2,4}= 12.
1000 = 83.12 4- 4 so a1000 = a4 = (138).
1.40 (0) We have that
(12)"1(135)-1(1579)(135)(12) = (12)(153)(1579)(135)(12)
= (5379)
which is of order 4.
Likewise, (579)"J(123)(579) = (123) is of order 3.
(b) The inverse of (135)(24) is clearly (153)(24). Thus
TOT-1 = (135)(24).(12)(3456).(153)(24) = (1652)(34).
1.41 First establish the result when r= 2; i.e. that for a transposition (A^A^) we
have that o(k1k2)o~1 = (o(k1)o(k2)). This is equivalent to
a{kik2) = (o(k1)o(k2))o;
and this equality results from the observation that swapping k\ and k2 then
applying a (i.e. the left hand side) gives the same result as first applying
G then swapping o(k{) and o(k2) (i.e. the right hand side).

50
Solutions to Chapter 1

For the general case, use the fact that every cycle can be written as
a product of transpositions:
(kXk2 •••*,) = (*i*2)(*2*3) ' ' ' (K-lKl
Thus the result follows from the observation that
a(ktk2 • • • kr)a~l = o(kik2)(k2k3) ••• {kr^kr)a-1
= o(k1k2)o~1 .o(k2k3)o~1 • • • o(kr—ikr) a"1.
It is readily verified that {(1), (12)(34), (13)(24), (14)(23)} is a subgroup
of S 4 . By the above, this subgroup is invariant under conjugation by a E S 4 ;
hence it is a normal subgroup.
1.42 Associativity of the operation on G\ x G2 follows from the associativity of
the group operations on both Gx and G2. The identity element is (1, 1) and
the inverse of (gug2) is (g\\g~2l).
It is easy to verify that Hi and H2 are subgroups of the cartesian product
group. They are in fact normal subgroups. For example, we have that
(x,yT\gi, l)(x,y) = (x-lglx,y-lly) = (x"1*,*, 1)
which shows that Hi is normal.
(Alternatively, it could be noted that the mapping # : Gi x G2 -* G2 des-
cribed by #Gfi, £2) —Si is a group morphism whose kernel is H\ which is
then a normal subgroup. Similarly, H2 is the kernel of the group morphism
</>: Gt x G2 -> Gi described by y(gi,g2) = gi.)
For the last part it suffices to note that
(gu 1)(1,*2) = (gugi) = (hg2)(gu 1).
1.43 (a) Not a morphism since [0]i 2 , the identity element of Z 1 2 , is not
mapped t o [0]i 2 .
(b) Since Cn is abelian we have

so / i s a group morphism. Writing C 12 = { 1 , a, a2,.. .,an} we have Ker / =

(c) This is a group morphism since


= ([x+y]2, [x+yU) = ([x]2+[y]2, [x]4+

= f(x)f{y).
Clearly, I m / = {(0,0), (1, 1), (0, 2), (1, 3)}a Z 4 and K e r / = 4Z.
(J) The mapping so defined is a group morphism since

51
Book 3: Groups, rings and fields

f([x]s+[y]8)=f([x+y]s)=[x
= M2+bl2=/(
I m / = Z 2 and Ker/ = {[2ft]8 | n E Z}.
(e) Not a morphism; for example,
/ ( ( I , *)(*, 1)) =/(A, *) = (12)(123) = (23)
b u t / ( I , *)/(*, 1) = (123)(12) = (13).
( / ) This is a morphism, for f(n<p) is easily seen to have the same effect on
each / as/(?r)/(^). Im/is the subgroup of S w + 1 consisting of all permutations
fixing ft + 1, and Ker / = {1}.
1.44 Consider a morphism / : Z - > ( D . Suppose that f(l) = q; then for every
ft E Z we have/(ft) = nq so that/is completely determined by / ( I ) .
1.45 G consists of the matrices
_H-ft -ft

L ft l + «
Observe that det An — \ so ^4W is invertible. Now observe by a simple matrix
multiplication \h.2XAmAn =Am + n. Since A0=I2 it follows t h a t ^ 1 = A_„EG,
whence G is a group.
Consider now the mapping # : Z -• G given by #(ft) = ^4W. This is a group
morphism since
f(m -\-ri\= A , = A A = f(ni\ f(ri\
Clearly, # is a bijection, whence it is an isomorphism.
As for the set G\ of matrices of the form
fl - 2ft ft "I
Bn=\
I-An 1 + 2ftJ
we note that det Bn = 1 so each Bn is invertible. Observe that BmBn =Bm+n
and that a similar argument to the above shows that Z and Gx are isomorphic
groups.
It follows that G and Gi are isomorphic.
1.46 By definition, G consists of the 2 x 2 matrices with entries 0 and 1 and with
determinant 1; i.e. G consists of the six matrices
ri oi ro n
Lo iJ' Li oJ'
i n ri on ro n ri n
o iJ' Li iJ' Li iJ' Li oJ'
52
Solutions to Chapter 1

Writing Xx= [0 l]\ X2=[\ 0 ] \ X3 = [1 i f we see that sends

Xi -+X2, X2 -+Xl9 X3 ->X3 and therefore corresponds to the permutation


(12). Similarly we see that
[1 11
corresponds to (13);
[1 01
corresponds to (23);
TO 1]
corresponds to (132);

corresponds to (123);

and/ 2 corresponds to the identity permutation. Clearly we have that G ^ S3.


1A7 Consider # : Z [X] ->(D+ given by
a0 + axX + • • • + anXn ->pgo p f 1 '"Pann
where Po<P\<"'<pn are the first n + 1 primes. That # is a group
morphism follows from the fact that if f(X) = ao+ a{X + V anXn and
m
g(X) = bo + bxX + h bmX then (assuming without loss of generality
that n < m)
P$° + b°Pi1 + b1' "Pn^^Pnlt1' ' 'Pmm
= (pS0Pl1"'Pnn)(P00Pl1"'Pbmm)

Suppose now that f(X) GKer ^. Then p^pf1 --p%n = l. But the pt are
distinct primes so we must have a0 = a^ — • • • = an = 0 whence f(X) = 0.
Thus Ker & = {0}. Also Im # = Q+ since, given any m/n E (D+ (as usual in
its lowest terms), we can express each of m and n as products of powers of
primes (using the fundamental theorem of arithmetic) and hence find a poly-
nomial that maps to m/n under #.
1.48 A simple matrix multiplication shows that A3 = 0. Now by definition we
have Ax = I3+xA+ ^x2A2 and so
A* Ay = (/3 +xA + WA2)(h +yA+ \y2A2)
= I3+yA+ y2A2 +xA+ xyA2 + %x2A2 (since A3 = 0)

53
Book 3: Groups, rings and fields

whence we see that G = {Ax | x G IR} is closed under multiplication. Now


A0=I3 is clearly the identity element of G; and AXA.X=AO shows that
Axl = A _x. Thus G is a group, which is abelian since AxAy = Ax+y= AyAx.
Consider now the mapping / : IR -+ G given by f(x) = Ax. This is a group
morphism since

Clearly, / is surjective. Now if x EKer / we have Ax = f(x) = / 3 and this


implies that x = 0 (since, for example, the (3, 3)th element of Ax is 1 4- \x2).
Thus we see that/is also injective, whence it is an isomorphism.
1.49 It is a routine matter to show that G is a group. The identity element is
(0, 1) and inverses are given by

G is not abelian; for example, (1, 2)(2, 1) = ( 5 , 2) whereas (2, 1)(1, 2) =


(3, 2).
Use the usual subgroup criterion to show that H, K, L are subgroups of
G. Of these, H and L are abelian.
HDK = {(0, b)\b>0}. Consider the mapping / : H n K - > IR + defined
b y / ( 0 , b) — b. This mapping is clearly a bijection. Moreover,
/ [(0, a)(0, b)] = / ( 0 , ah) = ab= / ( 0 , tf)/(0, b)
so that/is also a morphism. Hence / i s an isomorphism.
(a, b) has order 2 in G if and only if

For this to be the case we must have b = ±1. Now b — 1 gives 2a = 0 whence
# = 0 and we have the identity element; and if b=— 1 then we have
(a, - 1 ) 0 , - 1 ) = (0, 1) for every a G IR. Thus we see that {(a, - 1 ) U G IR}
is the subset of elements of order 2.
1.50 It is a routine matter to verify that the given operation is associative. (0, 0)
is the identity element, and the inverse of {a, b) is ((— l)b+1a, —b). Thus
G is a group, which is non-abelian since, for example, (0, 1)(1, 0) = (—1, 1)
and(l,0)(0, l) = ( l , l ) .
The mapping/: G -> G given by f(a, b) = (0, b) is a morphism since
/((*, b)(c, d)) =f(a + ( - l ) V b + d) = (0,2> + d),
/(*, 6)/(c, d) = (0,6)(0, d) = (0, b + d).
The kernel of/is # and so i / is a normal subgroup of G.

54
Solutions to Chapter 1

The image of / is K so K is a subgroup of G. However, K is not a normal


subgroup since, for example

By the first isomorphism theorem, we have K = I m / ^ G/Ker/= G/H.


1.51 It is easy to check that the given operation is associative. The element (e, 1)
is the identity for G, where e is the identity in A. The inverse of (a, e) is
(a~e, e) and so G is a group.
If G is abelian we must have abe = ba8 for all a, b GA and e, 5 = ±1 and
it is easy to see that this means we must have a2 = b2 = e in A. So for
(a) take y4 = {e} the trivial group. (Note that a cyclic group of order 2 (Z 2 )
or the Klein 4-group ( Z 2 x Z 2 ) would also do.)
For (b) take ^4 to be any group containing an element not of order 2; for
example,^ = Z 3 or^4 = Z.
It is easy to check that TV is a subgroup. To establish normality we
show that nGN, g G G =*gng'1 GTV : if n = (a, 1) and # = (6, e) then
g'1 = (b~e, e) and so the second component of gng'1 is e2 = 1 and so
gng^GN.
It is clear that TV is isomorphic to A under the assignment a -* (#, 1). It
is also clear that the mapping/: G -+E described by f(a, e) = e is a surjective
group morphism whose kernel is TV. Consequently we have that G/N^E.
For the last part, take A to be the multiplicative group of non-zero real
numbers. Define # : G-HR \ {0} by &(a, e) = ea. That # is a group morphism
follows from
#[(a, e)(b, 5)] = &(ab\ eS) = e8abe.
For, by considering separately the cases e = 1 and e = — 1, it is easy to see
that (with the notation of question 1.9) e8abe = (ea) * (8b). Clearly, # is
a bijection and hence is an isomorphism.
1.52 To prove that (xy)n =xnyn use induction : the equality clearly holds at the
anchor point n — 1; and for the inductive step observe that, since G is abelian,
(xy)"*1 = x(yx)ny = x(xy)ny = xxnyny = xn+1yn+1.
Suppose now that x, y GH. Then there exist m, n such that xm = 1 =yn.
Now, using the above, we have that
(xv~l)mn = xmn( y~l>)mn = ( x m y \( y~1Y2lm
= (xm)n[(yny1]m =l
and so xy'1 GH. Thus H is a subgroup.

55
Book 3: Groups, rings and fields

Consider now G = { z £ C | | z | = l}. Every element of G is of the form


e^1 where # G IR is determined to within a multiple of 2TT. NOW
i sin w# = 1
= Im-n for some integer m
# = air for some a G (Q.

We now observe that if em = e^ then e (d "~^ = 1 and so # - v? = 2nrr for


some « G Z and hence

Consequently we have that

7T 7T

and so we can define a mapping/: G -> IR/Q by /(e ) = — + (D. Since


7T

<D

-/(e«)
we see that/is a group morphism.
Now/is surjective since (Vr G IR)/(er7rl) = r 4- (D.
Finally

K e r / = (eddii II - ++ <Q<Q =0 04 4-Q(Q) 1 1


) ( =^ (e^1 1 - G <D)
I 7T / I 7T J

= H,
and by the first isomorphism theorem it now follows that
G / i / = G / K e r / ~ I m / = IR/GL
7.5J / f = {(1), (12)(34), (13)(24), (14)(23)}. Now check that for every heH
we have (123)"1/z(123) GH, so that His normal in G.
G/# is cyclic of order 3 generated by (123)//.

56
Solutions to Chapter 1

K is normal in H since H is abelian. But (12)(34) G K and


(123)"1(12)(34)(123) = (321)(12)(34)(123) = (13)(24)
so K is not normal in G.
1.54 ((25)(34))~1(12345)(25)(34) = (25)(34)(12345)(25)(34) = (54321) G H
since (54321) = (12345)"1. G/H is generated by (25)(34)# which has order
2 and so \G/H\ = 2. Hence | C | = \H\.\G/H\ = 5.2 = 10.

The rotational symmetries are generated by (12345). The only other


symmetries are reflections in lines joining the centre of the pentagon to the
mid-points of the edges (see question 1.11). Each of these can be obtained
by a rotation followed by the reflection (25)(34) and the inverse rotation.
Hence G is isomorphic to a group of symmetries. See Fig. SI.4.
1.55 Use the usual criterion to show that nX is a subgroup of raZ, and that
mZ/«Z is a subgroup of X/nX. To show that the given prescription defines
a map we must show that if a + nX = b + nX then/(a + nX) =f(b + nX).
But a +riX.= b + nX implies a — bGnX so a — bEmX since m divides
n. Hence a 4- mX — b + mX as required.
We now observe that
Ker/= {a + nXE: X/nX \ m divides a}
= mX/nX.
The first isomorphism theorem now gives
(X/nX) X
(mX/nX)~ mX
1.56 It is a routine task to show that A is associative (see Book 1, question 1.10).
Clearly, the identity element under A is 0 and EAE = 0, which shows that

57
Book 3: Groups, rings and fields

E~l = E and hence P(X) is a group. It is clear that every non-identity element
is of order 2.
If X— {1, 2} then |P(X)| = 4 and, since every non-identity element is
of order 2, the group cannot be cyclic. We must therefore have P ( X ) ^
C2 x C2.
1.57 That (pg:aH-^gaH is a bijection is immediate from the observation that
aH ->g~1aH is an inverse for it.
Now define a mapping # from G to the permutation group on S (which
is Sk since \S\ = k) by $(g) = ^ . Then &(gh) is the map aH -*ghaH, and
•&(g)&(h) is the map aH->g(haH). Consequently we see that # is amorphism.
(Note that if H = {1} then 5 ^ G and we obtain Cayley's theorem.)
To find Ker & look for elements of G which lead to the identity permuta-
tion. In particular, the coset H maps to gH which must be H and so g EH.
Thus we see that Ker # CH
If TV is a normal subgroup of G and NCH we show that TVCKer &. Let
nEN and consider &(ri). This is the map aH ->naH But na = aw' for some
n EN (since N is normal) and hence naH = an'H = aH since n' ENCH
Hence #(«) is the identity permutation and n E Ker # so that N C Ker #.
Suppose now that H contains no non-trivial normal subgroup of G. Then
Ker # = {1} and # is injective. Hence Im ^ ^ G and has order \G\. But
Im & is a subgroup of 5fe which has order k\. So, by Lagrange's theorem,
we have the contradiction that |G| divides k\.
For the last part we note that since the index of H in G is 9, and 99 does
not divide 9!, H contains a non-trivial normal subgroup of G. But H has
prime order and so by Lagrange's theorem the only non-trivial subgroup
of H is H itself. Hence H is a normal subgroup of G.
1.58 For each of the four groups we give a property which the group possesses
and which the other three groups do not. This shows that no two of the
groups can be isomorphic.
(a) Every element has order 2 (see question 1.56).
(b) The group is cyclic.
(c) The group is not abelian.
(d) The group is isomorphic to C4 x C2- For, 2 generates a cyclic
subgroup H of order 4, 11 generates a cyclic group K of order
2,HDK=l and the group is abelian.

58
Solutions to Chapter 1

1.59 The first part is routine.


(a) If / : Z ->• Z is an isomorphism then / ( I ) has a multiplicative inverse
in Z so /(1) = ±1. Hence Aut Z = {±1} under multiplication and so is
isomorphic to C2.
(b) Every morphism / : Z w -> Z w is determined by / ( I ) . As before, / ( I )
must be invertible i f / is to have an inverse. Using the euclidean algorithm,
we see t h a t / ( I ) must be coprime to n. The converse is also true, so / i s an
isomorphism if and only i f / ( I ) = k where k is coprime to n. Thus Aut Z w
is isomorphic to the multiplicative group of integers that are coprime to n.
(c) If G is the Klein 4-group then it is readily seen that every bijection
from G to G which takes the identity to itself is a morphism; this is because
G has the property that the product of any two non-identity elements is the
third non-identity element. Hence any permutation of the three non-identity
elements gives an automorphism of G. Thus Aut G^S3.
1.60 First we show that Z(G) = {Z*, Z * ( - l , 1)} so Z(G) has order 2. It is clear
that (—1,1) commutes with every element of H and so certainly we have

for all (x, y) EH. On the other hand, suppose that Z*(xu yi)GZ(G). Then
if (x2, y2) is anY element of it we have

where Z = (1, 1) or (—1,-1). In other words,


(xlx2,yly2) = (x2xuy2yi) or (-x
Now by symmetry we need consider only elements (xx, yx) of the form
(i, i), (i, - i ) , (i, j), (i, k), (i, 1). Note that (-i, - i ) = ( - 1 , - l ) ( i , i) and so
they are in the same coset relative to H and thus represent the same element
of G. We choose a specific (JC2, y2) for each of these five elements to show
that in each case Z*(xl5 y{) is not in the centre of G.
(1) (i, i)(j, l) = (ij, i) and (j, l)(i, 0 = (-ij, i) = ( - l , l)(ij, 1). But
( - 1 , l ) e z * andsoZ*(i,j)(j, l)#Z*(j, 1)0, j).
(2) (i, - i ) ( - j , l) = (-ij, - i ) and ( - j , 1)0, - 0 = ( - l , D(-ij, ~ 0 =

(3) (i, j)(j, 1) = (ij, j) and (j, 1)0, j) = (-rj, j) = ( - 1 , l)(ij, j).
(4) (i, k)(k, 1) = (ik, k) and (k, l)(i, k) = (-ik, k) = ( - 1 , l)(ik, k).
(5) (i, 1)0, 1) = (ij, 1) and (j, l)(i, 1) = (-ij, 1) = ( - 1 , l)(ij, 1).
This proves that Z(G) = {Z*, Z*(-l, 1)}.

59
Book 3: Groups, rings and fields

If we take a closer look, we can see that we have proved in addition


that G/Z(G) is abelian. Hence, if gl9 g2GG we have gxg2 —gig\Z where
zez(G).
Suppose that G = GxxG2. Then either GlC\Z(G) = l or Z(G) since
Z(G) has only two elements. Suppose without loss of generality that
Gi HZ(G) = 1. Then G\ is abelian. For, if x, y G G\ we have xy =yxz for
some z G Z(G) whence xyx~1y~1 G Z(G) n d = 1 and hence xy = JJX.
Finally, Gi = l. For, every element of Gx commutes with every element
of G2 (since G = Gtx G2) and hence Gx < Z(G). But Gt n Z(G) = 1 and
consequently Gi = 1.

1.61 Note that


S = {A G Mat 2x2 (IR) | det(X + / 2 ) =£ 0}.
Thus, if ^ , J? G 5 we have
0 ^ det(^ + / 2 ) det(B + / 2 )
= det[U+/2)(5+/2)]

and consequently A + B + AB G S.
To prove that * is associative we observe that

It follows that

Clearly, A * 0 = A and 0 * B = B and so 0 is the identity element for *. Now


A*B = 0<*A +B + AB = 0*>(A + I2)B = -A

so the inverse of A with respect to * is —(A 4- I2)~lA. Thus S is a group.


Consider now the mapping f:S->M given by f(A) =A+12. That / is
a morphism follows from the observation

If / ( 4 ) = f{B) then ^ +1 2 = £ + / 2 gives ,4 = 5, so / is injective. Finally,


if XGM then clearly X-I2E.S and f(X-12) = X, so that / i s also surjec-
tive. Thus/is a group isomorphism.

1.62 We have that

60
Solutions to Chapter 1

1 + xy
ol -\-x2y2-x2-y2>0
o(l-x2)(l-y2)>0.
Consequently, if JC, y G G then
x+v
-GG.
1+xy
x +y
For x * v = it is readily seen that
1+xy
x + y + z 4- xyz
X * O * z) = — ; ; = (X * y) * Z.
1 + xy + xz + yz
Thus * is associative. Clearly, * is commutative; and since JC * 0 = x it follows
that 0 is the identity element. Finally, x * (--JC) = 0 shows that — x is the
inverse of x under *. Hence G is a group.
1 +JC
With/(jc) = log we have
1 —x
x +y
1+ -
1+jcy 1 + xy +x +y
fix *y) = log •— = log —
x +y \+xy—x—y
1 +xy
/1+jc 1 + j A l +xy + x + y
f(x)+f(y) = log ( - ~ )) = logg -
\1 — JC 1 — yiyi l+xy
l — x—y
and so / i s a morphism.
That/is injective follows from the fact that
1+JC 1+JC
fix) = 0 => log = 0 => = 1 =• JC = 0.
1—JC 1—JC
1+JC
To see that / is also surjective, we observe that JC -> maps ]—1, 1 [ onto
1 —JC
IR+, and x -> log JC maps IR+ onto IR. Hence/is an isomorphism.
1.63 Suppose first that there is a law of composition * on H making H a group
with / an isomorphism. Then/" 1 : H-+G is also an isomorphism and hence

61
Book 3: Groups, rings and fields

(\/x,y<EH)f~l(x * y) = f~1(x)f~1(y) whence we have that


(\/x,yGH) x*y=f[r1(x)r1(y)].
Thus, if such a law * exists then it must be given by the above formula,
and hence is unique.
We must now show that such a law exists. The way to do so is clear:
define on H the law of composition * by

Then * is associative; for


x*(y*z) =f[rl(x)rl(y)f~l(z)] = (x * y) * z.
For every xGH we have x * / ( l ) =f[f~l(x) • 1] =x and similarly
f(l)*x=x. Thus / ( I ) is the identity of H. Given xGH, consider now

Thus / K/" 1 ^))" 1 ] is the inverse of x in H and so H is a group. The bijection


/ i s also an isomorphism since, for all x, y G G, we have
f(x)*f(y) =/[/"1(/W) -r\f(y))] =f(x -y).

62
Solutions to Chapter 2

2.1 (a) To show that S is a ring under + and • we check the appropriate
axioms:
(0 / + g is a mapping from IR to IR;
(ii) For every xGIR we have [(f+g) + h](x)= [ / + ( # +ft)](*)
and so (/4-#)4-ft = / + < > +ft);
(iii) Define 0 : IR -> IR by 0(x) = 0 for every x G IR. Then 0 + / =
/ + 0 = / f o r all m a p s / : IR-*IR;
(iv) Given / : IR -> IR define - / : IR -> IR by (-f)(x)=-f(x).
Then/+(-/) = ( - / ) + / = 0 ;
(v) (/+tf)(x) = (* + /)(*) for everyx G IR sof + g=g + f;
(vi) /* g is a mapping from IR to IR;
(vii) [(f'g) * h](x) =[f'(g- h)](x) for every x G IR and hence
(f'g)'h=f'(g'hy,
(viii) [ ( / + ^ ) -ft](x)= [f-h+g-h](x) for every JCGIR and so
( / + # ) *ft= / *ft+ ^ •ft.The other distributive law is similar.
It is clear that /' • g — g- f and so S is commutative. Also, the map 1 : IR -> IR
given by l(x) = 1 for every x G IR is the identity of S.
S is not a ring under + and o; for example, the distributive laws fail:
take g(x) = x, h(x) = -x and f(x) = x2 to obtain [ / o (g 4- ft)](l) = 0 and

(b) A simple check of axioms shows as in (a) above that S is a ring which
is clearly commutative and has an identity element, namely (1, 1). Now
(a, b)GS has a multiplicative inverse if and only if there exists (c, cfyGS
such that (a, b) • (c, d) = (1, 1), i.e. if and only if ac = 1 and bd = 1. Thus
we see that (#, Z>) has a multiplicative inverse if and only if a =£ 0 and b ^0.

63
Book 3: Groups, rings and fields

2.2 (a) To show that R(X) is a ring it suffices to observe that if


> + y2 y2
Xy2 x2
then A —BER(X) and ABER(X); then R(X) is a subring of the ring M.
[1 0] {x+y y~]
The identity element of R(X) is clearly . Suppose that
L0 U L —y xi
is a non-zero element of R(— 1) (so that x and y are not both zero). Then it
is readily seen that ifa = x2+xy+y2 then a ¥= 0 and this matrix is invertible
with inverse

a a

a/ a a
which belongs to R(— 1).
[1 11
InZ?(0) the matrix has no multiplicative inverse.

(b) To show that F is a subring of Af observe that


fx xl [y yl [x-y x-y~\
y y x-.y x-yi
and that
r* x - i p ylJ2xy 2xylF
Ix xi lyly yi
y I2xy 2xy\
[2 21
The identity element of F is I 1 . If x ¥* 0 then has multiplicative
L 2 2 -"
"1 1 '
4x 4x
inverse . The last part is explained by the fact that in F the
1 1
Ax 4JC.
identity element is not the 2 x 2 identity matrix I2 of M.
2.3 Suppose that x G Zw is not invertible and that x i=- 0. Then K x < « with
hcf {x, «} =£1. Thus there exists t with K ^ O which divides both x and
= at andn = ht where 1 < # < f t and Kb<n. Now

64
Solutions to Chapter 2

xb =atb =an = O (mod n)


so we have xb = 0 in Z w , whence x is a zero divisor.
2.4 That K is a sub ring is routine. That it is commutative follows from the
observation that
a —b~\ re —dl vac — bd —{be 4- ad) 1
[b a\[d c \~[bc + ad ac-bd J

[d c Mb a\
ri 01
and clearly is the identity of K.

Note that 0 and 1 are the only squares in Z 3 so for a , 5 G Z 3 we have

a 4- b — 0 only if a = b = 0. Hence a non-zero element of A"


LZ? a J
has inverse in K where t = (a2 4- b2)'1. Consequently, K is a field.
l-bt at]
There are three possibilities for each of a and Z>, so there are 3 x 3 = 9
elements in K. Hence J£ contains eight non-zero elements. It is a routine
[1 2]
matter to show that each of these is a power of , whence the result
follows.
2.5 In (a), (c) and (d) we have examples of integral domains, so that x(x 4-1) = 0
implies x = 0 or x + l = 0 whence x = 0 or x = — 1. Hence there are two
solutions. In (b) it is readily seen that 0, 2, 3, 5 are solutions, so there are
four solutions in Z 6 .
2.6 (a) a = 3 works : powers 3 , 9 , 2 7 , 1 3 , 5 , 1 5 , 1 1 , 3 3 , 3 1 , 2 5 , 7 , 2 1 , 2 9 , 19,
23,1.
(b) 0 2 = 0, I2 = 1, 172 = 17 and 182 = 18.
2.7 That E is a ring is routine. If (a, b) GiTis a zero divisor then there exists (c, d) GE
with (c, d) ^ (0, 0) and (a, b) <S> (c, J ) = (0, 0). Now ac + bd = be + ad = 0
so 02c 4- abd = 0 and &2c + bad = 0 whence a2c = Z>2c, so c(#2 — b2) = 0.
If c = 0 then from #c + M = Z>c 4- ad we obtain M = ad so if i? has no zero
divisors a = b (note that d ¥= 0 since c = 0 and (c, d) ¥= (0, 0)). If c =£ 0 then,
again using the fact that i? has no zero divisors, we deduce from c(a2 — b2) = 0

65
Book 3: Groups, rings and fields

that a2 — b2 = 0 and so (a — b)(a + b) = 0 which gives a = b or a=—b.


Thus we have that (a, a) and (#, —a) are the only zero divisors inE. Moreover,
E contains the zero divisors (a, a) and (a, —a) whatever the properties of R
since (a, a) ® (a, -a) = (0, 0).
2.8 We have that
JC2 = 1 ^ jt2 - 1 = 0 <> (* + 1)(JC - 1 ) = 0
and since Z) is an integral domain there are only two solutions, given by
x + 1 = 0 and x-1 = 0.
To prove that if p is a prime then ( p —1)! = —l(mod p) we observe first
that if p = 2 then the result is trivially true. Thus we can assume that p > 2,
so that p is odd. Now Z p is a field, and every field is an integral domain; so
there are only two solutions of x2 = 1 in Z p . Clearly, these are 1 andp — 1.
Consider now the product
1.2.3...(p-l)
of all the non-zero elements of Z p . In this we can pair off every element with
its inverse and thereby see that the product reduces to p — 1; for, I" 1 = 1,
the number of factors is even, and p — 1 is the only element of order 2.
Consequently it follows that (p — 1)! = —l(mod p).
2.9 First, note that * is associative; this follows from the fact that
(a*b)*c = a + b + c — ab — ac — be + abc = a * (b * c).
Now G is closed under * since if a, b EG then there exist u, wGE with
a * u = 0 and b * w = 0. Then, using the fact that * is associative, we see that
(tf*Z?)*(w*w) = 0 * Z ? * w * w
= a*0*u
=a*u
= 0.
Similarly, (w * u) * {a * b) = 0 and hence a * b EG. That G is a group under
* now follows from the fact that a*0 = a = 0*a,so that 0 is the identity
element, and inverses exist by the definition of G.
If e — a GE is invertible then there exists x, which can be written in the
form e — b, such that e = (e — a)x = (e — d){e — b). Expanding the right
hand side, we obtain from this a*b=a + b— ab — 0. Likewise, the equation
(e — b){e —a) = e gives b * a = 0 and hence a EG.
Conversely, if a EG then there exists bEG such that a * b = 0, whence
a + b —ab = 0 and so (e — d){e — b) = e, so that e — 0 is invertible in#.

66
Solutions to Chapter 2

2.10 That H is a subring of Mat 2X 2(^) *s routine. A non-zero element of//"has an


inverse in H if and only if it has an inverse in Mat 2x2 (C) and this inverse
belongs to H. Now we have that the inverse in Mat 2X 2(£) °f
I" a 4- bi c + dil
l-c + di a-bi\
is given by
•y
\a — b\ —c — d\\
a1
L c — d\ a + bi J
where a = a + b2 + c2 + d2 ¥= 0. Note that a = 0 only if a = b = c = d = 0.
2

This inverse is of the form


* + Z>*i c*

where a* = fl/a, Z?* = —b/oc, c* = —c/ot, d* = — d/a and therefore belongs to


H. Moreover, it is clear that every non-zero element of H has an inverse,
whence H has no zero divisors. But H is not a field since it fails to be com-
fo ii ro n
mutative; for example, the matrices and do not commute.
L-l OJ L i OJ
If X2 = -I2 then det X=±l and so a2 + b2 + c2 + d2 = ±1. The only
solutions fora,b,c,dEZ. are
<i = ± l , b = c = d = 0;

d = ±l, a = b = c=0.
We can eliminate the possibility fl = ±l, b = c = d = 0 since this gives
X = ±/ 2 so that X2 =I2. The remaining six solutions provide matrices that
satisfy X2 = —/2, namely
r-i on n on ro -n
Lo i J ' Lo - d ' L i o J '
ro n ro n ro - n
L-i oJ' Li oJ' v-i o J '
2.11 That f(o0)=f(a)f(0) is a straightforward calculation. If a has a multi-
plicative inverse j3 then from a/3 = 1 we obtain

67
Book 3: Groups, rings and fields

and hence f{ot) =/(j3) = 1. Now clearly a2 4- b2 = 1 implies a = ±l9b = 0 ox


0 = 0, b = ±l, so 1, —1, i, —i are the only elements with multiplicative
inverses.
If / ( a ) is prime then a = (3y gives / ( a ) =/(|Sy) =f(ff)f(y) so either
/(|8) = 1 oif(y) = 1. Hence either 0 or 7 has a multiplicative inverse.
Now there is no element a with / ( a ) = 3 since a2 4- Z>2 = 3 is impossible.
Thus 3 G Z[i] is irreducible; for /(3) = 9 and 3 = a0 would require /(3) =
/ ( a ) / ( 0 ) , so if/(a) =£ 1 #/(jJ) then/(a) = /(0) = 3.
/(43i - 1 9 ) = 2210 = 2.5.13.17.
43i — 19 = (1 + i)(2 4- i)(2 4- 3i)(4 - i) is a product of irreducibles (since
each of the factors a has/(a) a prime).
2.12 It is readily seen that ~ is reflexive and symmetric. To show that it is also
transitive, suppose that we have (r1s2—r2Si)t = 0 and (r2s3—r3s2)t' = 0.
Multiply the first of these by t's3, the second by tsl9 then add; the result is
(rts3 — r3S\)tt's2 = 0 where tt's2 G S. Thus ~ is also transitive.
That Rs is a commutative ring with an identity is routine. (It may help
if you write r/s instead of [(r, s)], because then the sums look very much
like the usual sums you get in fractions.)
(a) The mapping described by [(r, s)] -> r/s is an isomorphism.
(b) Writing [(r, s)] as r/s we have that if r/s =£ 0 then r =£ 0 and so s/r G Rs
and is the inverse of r/s.
(c) WhenjR = Z 6 and£= {2, 4} the set M of ordered pairs has 12elements.
However, the equivalence classes under ~ are
{(0,2), (0,4), (3, 2), (3, 4)}
{(1,2), (2, 4), (4, 2), (5, 4)}
{(4, 4), (2, 2), (1,4), (5, 2)}
and so Rs has order 3.
2.13 Note that p2 — pq 4- q2 = (p — q)2 4- pq. If this expression is 0 then we
have, on the one hand, pq = p2 4- q2 > 0 and, on the other hand,
pq = —(p—q)2 < 0 , Consequently pq = 0 and hence p2 + q2 = 0 so that

pP 1
5 consists of the matrices of the form v
f Q

L-P and
easy to check that S is closed under subtraction q-p\
1 where p,qE(&. It is
multiplication, hence is
a subring of Mat2X2(IR)- That S is a field follows from the fact that it is

68
Solutions to Chapter 2

commutative and, by the above observation,


q P
detf ]=p2-pq + q2
l-p q-p]
is 0 if and only if p = q = 0, so every non-zero element of S has an inverse
under multiplication which is in S.
2.14 Suppose that a + b\/p = Q. If a#0 then clearly b ^ 0 and then y/p =
—ab~lGF, a contradiction. Thus a + byJp = 0 implies a = b = 0. The
converse implication is obvious. It now follows that
a 4- b \Jp = c 4- d \jp <> (a ~ c) 4- (b — d) V P = 0
o a—c= 0= b—d
<>a = c,b —d.
If F[\/p] = {a + b\/p\a,beF} then

(a + b y/p)(c + dy/p) = (ac 4- bdp) + (ad + be) s/p G F[Vp],


and so F[s/p] is a sub ring of IR. The identity element of this ring is 1 4- 0 \/p\
and since, for a 4- b \/p ¥= 0, we have
\
=
a2—b2p a2—b2p
)
with a2 — Z?2p ^ 0 (otherwise \/p —\a/b\€zF, a contradiction), it follows that
every non-zero element of F[\Jp] has an inverse. Thus F[\/p] is a field.
If \Jm G(Q[VWL saY sjm — r + ssjn where r, s G Q , then we have
m = (r 4- s \Jri)2 = r2 4- S2H + 2rs Vw- Since m is a positive integer it
follows that r =£ 0 and s •=£ 0, whence the contradiction %/w = (m — r 2 — s2ri)j
2rs G Q. Thus we deduce that y/m ? (D[\/«]-
For the last part we observe that
S = {a + Z> V ^ 4- c V^ + ^V0w«) U , i , c , d G ( Q }

[V] where F
so S is a field.
2J5 That R is a ring is routine. To check that S is an ideal, let f,g^S. Then

69
Book 3: Groups, rings and fields

so f—gGS. Also, for every rER,

sorfES and/r E S. Hence S is an ideal ofR.


T is a sub ring of i? since if /, g E 7" then
D ( / - * ) ( 0 ) = ( D / - Dg)(0) = D/(0) - Efr(O) = 0,
D(&)(0) = (D/.* + /.D*)(0) = D/(0)*(0) +/(0)E*(0) = 0.
However, T is not an ideal of R since, for example, / : IR —> IR given by
(Vx E I R ) / ( X ) = 1 satisfies D/(0) = 0, and so / E T, butg : IR -> IR given by
(Vx E IR)#(x) = x is ini? and/# = g £ 71.
For the last part we note that since S and T are subrings so also is S f) 71
Also, iffGS n r t h e n D / ( 0 ) = 0 and/(0) = 0 and so, for everygGR,
D(fg)(0) - (Df.g + /.D^)(0) = D/(0)g(0) + f(0)Dg(0) = 0.
2.16 Suppose that a, bGA(X). Then ax = bx = 0 for all xEX and so (a-b)x = 0
for all xEX, whence a-b eA(X). Also, if r<ER then (ra)jc = r(ax) = 0
for all x E X and so ra E ^(X). Thus ^4(X) is a left ideal.
Suppose now that c, dEB(X). Then xc — xd = 0 for all x E X and so
x(c-~d) = 0 for all x E X , whence c-dGB(X). Also, if rEi? thenx(cs) =
(xc)s = 0 for all x E X and so cs E £(X). Thus £(X) is a right ideal.
Suppose now that X is a left ideal. Let a E^4(X) and rGR. Then for every
x E X we have (ar)x = a(rx) = 0 since rx EX. Thus #r E ^ ( X ) and A(X) is
also a right ideal.
For the second part of the question we begin by noting that the given

set X of matrices is closed under subtraction. If now | | E R then


ic d\
ra 61 |~x 01 Tax + by 01
[c d\[yVy Oi [ex+ dy OJ
and so X is a left ideal.
To calculate A(X): for all x,y we must have
bivx oi ro oi
OJ LO OJ
i.e., ax + by = 0 = ex + dy. Taking x = 1, y — 0 then x = 0, y = 1 we obtain
a = b = c = d = 0. Thus ^4(X) consists only of the zero matrix.
To calculate B(X): for all x , y we must have

70
Solutions to Chapter 2

\x 01ra biro 0
Iv O j L d J L O 0
i.e., ax = bx = ay = by = 0, which gives a = b = 0. Thus B{X) consists of
the matrices of the form


lc
where c,dG IR.
Note that B{X) is not a left ideal since, for example,
p
Lo
2.17 It is given that \Jl={xGR \ Qn> l)xnGI}. Given x, yGyJl, suppose
n m n+m
that x Gl and y GL Consider (x— y) : every term in its binomial
expansion contains either xn or ym as a factor. Thus, since / is an ideal, we
have (x— y)n+m GI whence x—yG\Jl. Also, if rGR then, since R is
commutative, (rjc)" = r"xn E / whence rx G \/L Thus V^ is a n ideal of R.
If xGI then x = x1 GI gives x E y/l; so / C yjl.
By the above, we have that \Jl C >/(>//). To obtain the reverse inclusion,
suppose that xG^(\/l). Then we have xn Gy/l for some «, whence xnm GI
for some n, m. Consequently we see that x G yjL
We now note that ICJ=>\/lC\/j. In fact, if xGy/l then, for some
n
n,x GlCJ givesx G\Jj. To establish (a) we observe that

As for (Z?), it is clear from the above observation that \Z(IdJ')CI and
\J(I n / ) C \/«^ and so V(/ n / ) C V / ^ V^« To obtain the reverse inclusion,
let xGy/ldy/j. Then there exist «, m such that JCW E / and xm E / . Since
/ a n d / a r e ideals it follows t h a t x n + m = xnxm GIHJ, whencex E y / ( i n j ) .

2.18 That R is a subring of Mat 2X 20 R ) is routine. Also, simple matrix multi-


plication will show that R is commutative. The identity element of R is
the 2 x 2 identity matrix I2 of Mat2X2(IR)-
Again, simple computations show that if A, B E / then A — B E / , and if
XGR thenZ4 E/. Thus we see that / i s an ideal of R.
l
r V5i
Taking JC = 1, j> = 0 we see that E / ; and taking x = 3,

71
Book 3: Groups, rings and fields

T3 01
y - —1 we see that <E /.
LO 3J
LO 3J
Suppose, by way of obtaining a contradiction, that /=(>). Then for
some t,uERwehave
r i V5] [3 oi
= rt, \ \=m.
L-%/5 1J LO 3J
Then 6 = det r det t and 9 = det r det u give det r E {1, 3}, since for
a b 5
F \/ ~\ o 9
r=\ . \ GR we have det r = a + 56 which is an integer.
2 2 2 2
Now a + 5b = 3 is impossible; and a + 5Z> = 1 implies that r is invertible,
whence the contradiction / = (r) = i?.
2iP (0) False. See the example in (c) below.
(b) True. Let A, B be left ideals with al9 a2 GA and bu b2 GA Then
we have
Oi + b1)-(a2 + Z>2) = Oi -a2) + (&! - Z?2) GA
and, for any x GR,

\a 01
(c) False. Take ^4 to be the set of matrices of the form where
a ,1
a,b E IR, and 5 to be the set of matrices of the form where a,b E IR.
0 OJ
Lo oJ
Then A is a left ideal and i? is aright ideal of Mat2X20F0- But ^4 + B consists

c OJ
[ a b~\
where a, b, c E IR, and this is not a sub-

ring of Mat 2X 2(IR) since, for example,


ro oi ro l i ro 01
\$A+B.
Li oJ Lo oJ Lo lJ
(d) True. See (b) above.
2.20 That Hom(^4) is a ring is a routine verification of the axioms. Note that in
this case / o (g 4- /*) = ( / o ^-) + ( / o h) follows from the fact that / is
a group morphism.
If A — Z then every morphism / : Z -> Z is determined by / ( I ) , since
/(w) = / ( l + 1 + • • • + 1) = / ( l ) + * * * + / ( 1 ) = nf{\). The mapping & :
Hom(Z) -> Z described by # ( / ) ~ / ( l ) is then a ring isomorphism, for it is

72
Solutions to Chapter 2
=
a bijection and, on the one hand, ( / + # ) ( ! ) / 0 ) + £"(1) gi yes

whereas, on the other hand, if / ( I ) = p and#(l) = q then ( / o g)(l) — pq —


(g ° / ) ( ! ) and consequently

Exactly the same argument holds in the case where A = Z w .


Suppose now that A = Z x Z. If / , # : ^4 -*A are given b y / ( x , 7 ) = (x, 0)
and g(*, >>) = (x, x) then we have ( / o g)(x, y) = (x, 0) and (g o f)(x, y) =
(x,x), so t h a t / o g^g o/andhence Hom(^4) is not commutative.
2.21 (a) True. If ab = ba then /(aft) =/(6a) so f(a)f(b) =f(b)f(a). Since
every element of S is of the form/(0) for some a GR, the result follows.
(Z>) True. The identity element 1 of R satisfies \a~a=a\ for allflGi?
s o / ( l ) / ( a ) = / ( a ) =/(flr)/(l), showing t h a t / ( I ) is the identity of 5.
(c) True. Use (Z?) and the uniqueness of the identity in S.
(d) False. For example, Z6 has zero divisors and there is a surjective ring
morphism/: Z 6 -> Z 2 given by/(x) = A: (mod 2).
(e) False. Consider the example in (d); Z is an integral domain but
Z 6 is not.
( / ) True. If f:R->S is a ring morphism then K e r / i s an ideal of R, so
if R is a field we have Ker / = {0} or Ker / = R. Now we cannot have the
latter since this would imply that S is the trivial ring. Thus K e r / = {0} and
/ i s an isomorphism.

2.22 It is clear that/(x) + / O ) = f(x +y) for alljCj.y IR. Also,


y y ixy
[lx l x\ .y y. \xy\kxy
shows that/(x)/(.y) =f(xy) for allx,.y G IR. Thus/is a ring morphism.
Now f(x) =f(y) gives x = y and so / i s injective. Consequently we have
= I m / ^ IR.
The mappings : Mat2X2(!R)~> 'R described by

is not a ring morphism;


p for example, we have that

[' 'If2 'H 3

Ll lJ Ll 2J L 3

73
Book 3: Groups, rings and fields

[1 11 \2 11
but -> 2 and -> 4.
Ll lJ Ll l\
The mapping g of is the identity map on IR and so is an isomorphism.
However,/o g is not a ring morphism, nor is it injective or surjective.
la \
2.23 We have R = {- \a, b G Z, b £ 3Z} and the identities

c ad —be a c ac
b d bd b d bd
Since 3 is prime, if 3 does not divide b and 3 does not divide d then 3 does
not divide the product bd. It follows immediately that R is a subring of (Q.

If / = (- ER | 3 divides a ) then it is clear from the above identities that


\b )
/is an ideal of/?.
a
Consider a non-zero element - 4- / of the quotient ring R/I. We have
b

-ER with - £ /. It follows that - G R and - £ /. Now


b b a a
(a \/b \ ab
\b !\a I ba
a
and so - 4- / is invertible in R/I. Hence R/I is a field.

2.24 Since x—x2 for every xERwe have, for every aER,
a-\- a = (a + a)(a + a):=a2 + a2 + a2 + a2 =a +a + a + a.
Consequently we have that 2a = 0. Also, for all x9 y E R, we have
* + y — (x "*".y)(* + >0 = * 2 + xy + ^ + .y2 =x + xy +yx +y9
which gives x.y = —yx. But 2^x = 0 so yx = —yx and we deduce that
xy —yx, whence/? is commutative.
That A is associative is quickly seen by examining the Venn diagrams for
AA(BAC) and C4A#)AC; they are each as shown in Fig. S2.1.
It is now clear that P(5) is an abelian group under A, the identity
element being 0 and A being its own inverse. To check the distributive law
Cn(AAB) = (CCiA)L(CCiB), consider again the Venn diagram for each
side; in each case it is as shown in Fig. S2.2.

74
Solutions to Chapter 2

Fig.S2.1

Fig.S2.2

It is now clear that P(5) is a ring, which is boolean since AC\A— A for
every AE?(S).
Consider now the mapping fE : P(5) ->P(5) given by fE(X) = XDEf.
We have
fE(XAY) = (XAY) nE' = (XnE')A(YDE1)

=fE(X)nfE(Y).
- is a ring morphism. Clearly, Im fE = f*(Ef), and
Ker fE ={XG ?(S) \XnEf = 0} = ?(E).
Hence P(£*) is an ideal of PCS') and, by the first isomorphism theorem for
rings,

75
Book 3: Groups, rings and fields

2.25 We have that


/(o) o -ir*(o) o

r /(o)*(o> o -I
L Df(O)g(O) + /(0)D#(0) /(O)#(O)J
f 030(0) o

= <*(/£),
and clearly a(f) + a(g) = a(f + g). Hence a is a ring morphism.
[a 01
Im a consists of the matrices of the form where a, b G IR; and
lb a]
Ker/= {/G IR[X] | 0 =/(0) = D/(0)} = (X2).
2.26 That R and S are subrings is routine. Now R contains an element OL with
a2 = 2; for ( 0 + 1 \/2) 2 = 2. Thus if / : R ->S were an isomorphism we
would have
/ ( a 2 ) = / ( 2 ) = / ( l + 1) = / ( l ) + / ( 1 ) = 1 + 1 = 2
whence [/(a)] 2 = 2.
But S contains no element ft with /32 = 2. For, if (# + b y/3)2 = 2 then
a + 3Z>2 + 2ab y/3 — 2 and consequently ab = 0, so that either a = 0 or
2

Z> = 0. If a = 0 then 3Z>2 = 2, which is impossible since b G Z; and if & = 0


then a2 = 2, which is also impossible since a EX.
We conclude that no such isomorphism exists.
2.27 We have that (2n + I) 2 = An2 + 4« + 1 and so #[(2/i + I) 2 ] = 1.
Every member of the given sequence is of the form 100* + 11 for some
* G Z. But
#(100* +11) = #(100)#(*) + #(11) = 0.#(*) + 3 = 3.
Hence no member of the sequence is an odd square and therefore no member
can be a square.
2.28 The only two rings with p elements (where p is prime) must each have addi-
tive group Zp and either zero multiplication or the usual multiplication in
Zp. To prove that these are the only possibilities, let#be an additive generator
of Zp and suppose that the multiplication is not the zero multiplication, so
that g2 = kg for some integer * =£ 0. Then we can find an integer m such that

16
Solutions to Chapter 2

km = 1 (mod p). Consider now the element h = mg = g + g+--- + g


(m summands). We have h2 = m2g2 = m2kg = mg = h. If we call the ring
Rp then we can consider the mapping / from the ring Z p to the ring Rp
described by f(ri) = nh. We have
f(n + m) = (n + m)h = nh + mh\
finni) — {nm)h = nmh2 = nhmh
and so / is a ring morphism. Since every non-zero element of the group
Zp is an additive generator of Z p , this is also true of h. Consequently, / is
surjective. Since the sets in question are finite and have the same number
of elements, / is also injective. Hence/is an isomorphism and the multipli-
cation in Rp is the usual multiplication.
2.29 (a) True. If R is a subring of a field F and a, b GR with ab = 0 then
a, b GFwithab = 0, so a = 0 or b = 0.
(b) False. For example, Z is an integral domain but the quotient ring
Z/4Z = Z 4 is not an integral domain.
(c) False. For example, (D is an integral domain in which every non-zero
element is invertible. But Q \ {0} cannot be cyclic, since if it were generated
by r E (Q\ {0} we would have f1 = 1 for some n, which would imply that
r = ±1, which is impossible.
(d) True. If R satisfies the given property then R \ {0} is generated by
a as a multiplicative group. Thus, given x9 yGR\{0}, we have x—an,
m
y=a for some n, m and so xy—yx. Since 0 trivially commutes with
every element of R, it follows that R is commutative. Now R has an identity,
so an = 1 for some n since jR \ {0} is cyclic. Therefore every non-zero element
of R is invertible and so R is a field.
(e) True. Take, for example, R — F.
2.30 (a) If am = 0 and bn = 0, consider (a - b)m +n. Since R is commutative,
this can be expanded by the binomial theorem. Each term of the expansion
contains a power of a no less than m or a power of b no less than n. Hence
each term is zero and so (a — b)n+m = 0. Thus we see that if TV denotes the
set of nilpotent elements then a, b G N implies a — bGN. Also, if a G TV then
am = 0 implies, since R is commutative, that (ra)m = 0 for every rGR.
Hence raGN and so TV is an ideal.
(b) Suppose that r + TV is nilpotent in R/N. Then we have, for some
m, (r+TV)m=TV. It follows that rm+N = N and so rmGTV, whence

77
Book 3: "Groups, rings and fields

rmn = (rm)n = O for some n, and consequently rEN. Thus r + N is the


zero of R/N.
(c) First try some examples. If n = 6 then N= {0}; if « = 4 then
7V= {0, 2}; if n = 12 then TV = {0, 6}. In general, if
n=pV...pfr
with each pt a prime and each a,- > 1, then

ro n ro on
(J) The matrices and are both nilpotent (each satisfies
L0 Oj l l OJ
x2 = 0), but their sum is not nilpotent. Thus N is not necessarily an ideal
when/? is non-commutative.
2.31 Consider the rectangular array (with m 4-1 rows and n + 1 columns) whose
(/,/)th position is occupied by ajbjXl+i = a^xi.bjxL Then

is the sum of all the elements in this array. Now extend the array to an array
of size (m 4- n 4-1) x (m + n + 1) by defining at = bj = 0 for / > n 4-1 and
/ > m 4- 1. Then
n-Vm i k \

Z E «*-/»/
k= 0 \ / = 0 /
is the sum of the elements in the second array, this being achieved by
summing its SW-NE diagonals. Clearly, because of the zero entries in the
second array, the sums are the same.
To show that $x is a ring morphism it is clearly enough to verify that
it preserves products. Now if

then the product f(X)g(X) is, by definition, the polynomial


n+
n +m
m // kk
I ( I «*-/M
(
fc = 0 \ / = 0
The result is then immediate from the previous identity.
$x is surjective since for every a G IR we have
a = $x(a + OX 4- • • • 4-

78
Solutions to Chapter 2

Now /(X)GKer &. if and only if X-x divides / ( X ) . Thus Ker fc. is the
ideal generated by I - x (i.e. it is the smallest ideal containing the poly-
nomial X — x).
2.32 It is clear that &m preserves sums. As for products, we have, for f{X) =
ao+a1X+ - - + anXn mdg(X) = b0 +

f ^ f £ akHb\ ' xk]


»m(f(X)g(X)) = *« L*=o\/=o J

n+p / k

n+p t k
( (**-/)*m(*/)

\/=o / V=o

Thus i^m is a ring morphism. Since ^ m is surjective, so also is &m. The kernel
of £ m consists of all polynomials 2f =0 #/X* such that $m{ai) = 0 in Z m for
every z; i.e. of all polynomials of the form m(Zf=o^iXl).
(a) Taking coefficients in Z 5 we obtain, by long division,
X 4 + 3X3 + 2X2 + X + 4 = (3X2 + 2X)(2X2 + 3X + 2)
-3X+4
so the quotient is 2X2 + 3X + 2 and the remainder is - 3 X 4- 4 = 2X + 4.
(b) InZ[X]wehave
X10 + 1 = (X2 + 1)(X8 - X 6 + X 4 - X 2 + 1).
Applying the morphism £2 w e obtain, in Z 2 [X],
X 10 + 1 = (X2 + 1)(X8 +X 6 + X 4 + X2 + 1).
Hence the quotient is X 8 4- X6 + X 4 + X2 + 1 and the remainder is - 1 = 1.
For the last part of the question we note that in Z [X] we have
X5 - 10X + 12 = (X3 - 2X)(X2 + 2) - 6X 4-12.
Applying the morphism &n to this, we see that X2 4- 2 divides X5 — 10X + 12
in Z w [X] if and only if - 6 X 4-12 = 0 in ~Ln [X]. This is the case if and only
if 6 = 0 = 12(modft);i.e. if and only if n — 2, 3 or 6.

79
Book 3: Groups, rings and fields

2.33 (a) Z 6 [X] has zero divisors; for example, 2X.3X=0 but 2X=£0 and
3X ¥= 0. If / ( X ) = 1 + axX + • • • + anXn then / ( X ) cannot be a zero divisor;
for, if g(X) = b\X 4- h bmXm has constant term zero then
f(X)g(X) = bxX + • • • + anbmXn+n * 0.
If / ( X ) has constant term # where a is not a zero divisor in R then / ( X ) is
not a zero divisor in R [X].
(b) If 1 +aX is invertible then there exists b0 + bxX+ • • • 4- Z>mXm GjR [X]
with
(1 4- aX)(b0 + b1X+--- + bmXm) = 1•
Thus we have that
b0 + (bx + abo)X + (Z?2 + ^ O X 2 + • • • + (bm

from which we obtain


bo = h h = -a, b2 = a2, b3 = -a3, ...,bm = ±am,
and abm = 0.
Then abm = 0 gives am + 1 = 0.
Conversely, if #w = 0 for some n then it is easy to check that
1 - a X + a2X2 - a3X3 4- • • • ± a " " ^ " 1
is the inverse of 1 4- aX.
(c) If F is a field then the non-zero constant polynomials are the only
invertible elements of F[X]\ for we have
deg (f(X)g(X)) = deg/(X) + degs(X)
and so deg(/(X)#(X)) = 0 only if deg/(X) = deg#(X) = 0.
2.34 Let p(X) = anXn 4- • • • + a0 and q(X) = bmXm +• • • + b0 with an ¥= 0 and
Z? m ^0. Then p(X)q(X) =anbmXn+m 4- • •• + aobo and, since ,R is an
integral domain, anbm ~=h 0. Hence the product of two non-zero polynomials
is non-zero and so R [X] is an integral domain.
Consider now the polynomial X. If X had a multiplicative inverse we
would have
anXn + 1 + • • • + a0X = X(anXn + • • • + a0) = 1
which is impossible. Hence R [X] is never a field.
Now let R = Z 4 . Suppose that the polynomial 0wXn 4- h # 0 has inverse
bmXm 4- h b0. Then we must have aobo = 1 and so, since 1 and 3 are the
only elements in Z 4 with multiplicative inverses, it follows that either
00 = 1 or #o = 3.

80
Solutions to Chapter 2

If p(x) = anXn 4- • • • + a0 is such that a0 = 1 or 3 and, for / > 1, at = 0


or 2 then we can write p(X) in the form 0O 4- 2q(X) for some g(X) G Z 4 [X].
Since in Z 4 we have I2 = 3 2 = 1 and 4 = 0, it follows that
[p(r>] 2 = («o + 2q{X)f =al+4aoq(X) + 4[q(X)]2 = 1.
Hence p(X) is invertible.
2.55 (a) Let f(X) = X2 and g(X) = 2X in Z 4 [X]. Suppose that

Then we must have deg r(X) < 1 and so r(X) is a constant polynomial. But
2X2 = 2r(X), which provides a contradiction.
(b) Let/(AT) = X2 and#(X) = 2X in Z[X]. Again suppose that

Then, since Z is an integral domain, deg q(X) = 1 and so

which gives 2a = 1, a contradiction since the coefficients are in Z.


2.56 Use the euclidean algorithm:
(a) hcf = X - l ; « ( X ) = K ^ 2 - 3 X + 3 )
(6) hcf = X - i ; f l ( X ) = -X;6(AT) = 1;
(c) hcf = X2

l
2.57 First observe that if K
K = I| | then
L—l o
K2=K-I2.
Now (al2 + 6A") - (cl2 + d^) = (a - c)I2 + (6 - rf)^ and so F is stable under
subtraction. Also, using the above observation, we have
(al2 4- bK)(d2 + dK) = acl2 + adK + Z)c^ + bdK2
= ac/2 +adK + bcK + bdK-bdI2
= (ac - bd)I2 + (ad + be 4- M)i^
so that Fis also stable under multiplication. HenceFis a subring of Mat2X2(IR)-
Swapping a, c and b, d in the above multiplication, we see that F is com-
mutative. Now
[a 01 \b bl [a + b b~\
Lo a\ Y-b OJ l-b a\
and the determinant of this is a2 + ab 4- b2. We now observe that

81
Book 3: Groups, rings and fields

(In fact, if a2 4- ab 4- b2 = 0 then we have on the one hand ab = — a2 —


b2 < 0, and on the other hand ab = (0 4- b)2 > 0 so that ab = 0 and hence
0 = 0 = b.) From this observation it follows that al2 + bK has zero deter-
minant (i.e. is not invertible) if and only if a = 0 = b. Thus we see that every
non-zero element of F has an inverse inF, so F is a sub field of Mat2 x2('R)-
There is no q E (D such that q2 — q 4- 1 = 0 since the roots of x2 — x +
1 = 0 are complex. ThusX 2 — X 4-1 is irreducible over (Q.
Now (&[X]I(X2 — X + 1) is a field which can be regarded as {a + bx |
0, Z?GQ, x 2 = x — 1 } . This is clearly isomorphic to Funder the assignment
a + bx*>aI2 + bK, since£ 2 =K-I2.

2.38 That /? forms a subring of Mat 3x3 (IR) is routine. That a is a ring morphism
is also straightforward. Clearly, a is surjective. By the first isomorphism
theorem we have that R/Ker a ^ I m a . The required ideal is therefore Ker a,
which consists of the matrices of the form

0 a b
0 0 c
.0 0 0

where a, b,cG IR.

2.39 The product is

6a 4- 3b - 2c - d 3a + 3b—c — d 3a 4- 6b - c — 2d
—10a — 5b + 2c + d —5a-5b+c +d -5a—10b
2a + b + 2c + d a+b +

It follows that

b
€ K :B ' 3 -1"
b
(t: K %:;
-5
1
1
1

82
Solutions to Chapter 2

Also, since

p • 41, ",'U2 °i
Li 1 2J Lo 2J
it is similarly seen that # preserves multiplication. Hence # is a ring morphism.

Now E Ker # gives

"3 -l"
-5 1
Lc dill 1 2J
1 1
' 3 -1
[2 1 11
Multiplying on the left by and on the right by - 5
pro- 1

\a
[a bbl 1 1 .
duces = 0, from which we deduce that Ker # ={0}.By the first iso-
Lc d\
morphism theorem we then have that I m ^ ^ Mat 2X 2(^)-
2.40 We have that
X5 + X 4 + 2X3 + 2X2 + 2X+ 1 = {X4- ^(X 4 + X 2 + 1)
+ X3+X2+X
X4 + X2 + 1 = (X - 1) (X3 + X2 + X) + X2 + X + 1

Hence/z=X2+X+l.
Clearly / , g G {ah \ a E Q[X]}. But any ideal containing / and g contains
h since
X 2 + X + 1 = X 2 ( X 4 + X 2 + 1)
4- (1 - X ) ( X 5 4- X 4 + 2X3 4- 2X2 +2X4-1).
Consider the mapping f : (D[X]-*C induced by substituting e 2?ri/3 for X.
Clearly,Kerf = / . Also
( / 2TT 2TT\ \
Im f = j p + # I c °s f" i sin — I | p, q E (D j

h +4 i|p^E

The result now follows by the first isomorphism theorem.

83
Book 3: Groups, rings and fields

2.41 Working in Z 5 [X] we have


/ = X44-X34-4X2+X-2 f-X3+X2 ~2=g
4 3
Xg = X + X -2X -*•--'
f-Xg = 4X 4- 3X» 2 —--4X 3 + 3X2 - 2X
2
= X(f-Xg)
2
4X -2X-2=g + X(f-
0
The highest common factor of/(X), g(X) in Z S [X] is thus 4"1(4X2 -
2X — 2). Now in Z 5 we have 4"1 = 4, so the highest common factor is
16X2 - 8 X - 8 = X2 - 3 X - 3 = X 2 4- 2X + 2.
Working in Z 7 [X] we have
/ = X 4 + X 3 + 4X2 + 4 X - 2 r—-X 3 +X 2 + 5X -2=g
Xg = X4 +X 3 4- 5X2 - 2 X - - - - '
f-Xg= -X 2 + 6X»2 - - - X 3 + 6X 2 -2X
6X2-4X +- 3X-2
3X-2 +- 3X-2
0
The highest common factor of/(X), #(X) in Z 7 [X] is thus 3" 1 (3X- 2).
Now in Z 7 we have 3"1 = 5, so the highest common factor is
5(3X - 2 ) = 15X-10 = X - 3 = X + 4 .
TO q
2.42 For the matrix K = \ ^ ^ | we have that K2 = ql2. Thus
= il 0
(al2 + bK)(cI2 + dK) = acl2 + adK 4- bcK 4- bdK2
= acl2 4- adK 4- bcK 4- bdql2
= (ac 4- bdq)I2 4- ( ^ 4- bc)K.
This shows that ^ is stable under multiplication. It is also stable under
subtraction since
al2 + bK-(d2 4- dK) = (a - c ) / 2 + (b-d)K.
Thus we see that iS^ is a subring of Mat2X2(®)- Clearly, I2 is the identity
element of Sq\ and from the above multiplication we see that Sq is
commutative.
Now
, \ a 01 TO qlja bql
lo a\ Ll OJ [b a\

84
Solutions to Chapter 2

and so al2 + bK = 0 if and only if a = b = 0. Also,


det(fl/2 + 67Q = a2 ~ Z>2#
and this is zero if and only if a = b \/q. Thus we see that every non-zero
element of Sq has an inverse (i.e. Sq is a field) if and only if V<7 € (Q.
If V<7 £ ® then clearly X2 —q is irreducible over (D and consequently
(D[X]/(X2 — <7) is a field. This field can be regarded as
{a + bx\a,be<h,x2 = q}.
Clearly, since K2 = ql2, this is isomorphic to the field Sq under the assign-
ment a + bx<*aI2 + bK.
2.43 Suppose that x G R is such that x'1 exists. If x GI then we have 1 = x~lx GI
and then(VrGR)r = r\GIso thatR = I.
It is clear that each of/i,/ 2 ,/ 3 is stable under subtraction and multiplication,
so that each is a subring of R.
Consider now the general products
a bl pc yl [ax ay + bz\
[
0 cj Lo zj Lo cz 1
[ x yi Va bl

Taking z 0= 0zjLO
we see that
Vxa xb + yC]
zc J
CJ ~theLOproducts are in Ix and so /x is an ideal ofR.
Taking x = Owe see that the products are in / 2 and so 72 is an ideal of R.
Taking x = z = 0 we see that the products are in 73 and so 73 is an ideal of R.
Suppose now that 7 is an ideal of R with 7^= {0}. If 7 contains a matrix of
[x yl
the form with x =£ 0 and z =fc 0 then, since this matrix is invertible in
LO z\
R, it follows by the first part of the question that 7 = 7?.
[ x

0 OJ
y~\
with x =£0
then 7 contains all left/right multiples of this by elements of R, whence 7 con-
x y~\
[
0 zj
in 7 with x, z =£ 0 whence the

contradiction I = R. Hence in this case we have 7 = Ih A similar argument


[0 yl
shows that if 7 contains with z ^ O then 7 contains 72 and in fact
LO Z J
coincides with 72. Clearly, the only other possibility for 7 (other than {0},
R,IhI2)isI = h.
85
Book 3: Groups, rings and fields

2.44 Since we are dealing with Z 2 , all coefficients are either 0 or 1. If/(X) has an
even number of non-zero coefficients then 1 is a root of /(X) = 0, so X 4- 1 is
a factor of f(X). Thus, if f(X) is irreducible then it must have an odd number
of non-zero coefficients. Clearly X is irreducible of degree 1. Also, any poly-
nomial of degree greater than or equal to 2 with zero constant term is not
irreducible. Consider now polynomials with constant term 1.
There is only one irreducible polynomial in Z 2 [X] of degree 1, namely
X 4- 1. Hence the only polynomial of degree 2 that is not irreducible is
(X + 1)(X + 1) = X2 4-1. Hence X2 4- X 4- 1 is the only irreducible of degree
2. Similarly, (X2 4- X 4- 1)(X 4- 1) and (X 4-1)3 are the only polynomials of
degree 3 that fail to be irreducible, so X3 4- X 4-1 and X3 4- X2 4-1 are the
irreducibles of degree 3. Likewise,
I 4 + I + l , X 4 + X 3 + l, I4+I3+I2+I+l
are the irreducibles of degree 4.
Examples of fields with 4,8,16 elements are Z 2 [X]/(p(X)) where p(X) is
I 2 + I + 1 , I 3 + I + 1 , I 4 + I + 1 respectively.
The six given polynomials all have degree less than or equal to 8, so are
either irreducible or have a factor which is irreducible and of degree less than
or equal to 4. We obtain
(a) X 5 +X 4 4-1 = (X 2 +X+1)(X 3 4-X+1);
(b) X6 4- X 5 4- X2 4- X + 1 is irreducible;
(c) X6 4- X 4 4- X3 4- X2 4- 1 = (X2 4- X 4- 1)(X4 4- X 3 4- X2 4-
X+l);
(J) X 7 4- X6 + X 4 4- X 3 + 1 = (X3 4- X 4- 1)(X4 4- X 3 + X 2 4-
X+l);
0 ) X 7 4- X 6 4- X 3 4- X 4-1 is irreducible;
( / ) X 8 4- X 6 4- X 5 + X 4 + X3 4- X2 4- 1 = (X2 + X 4- 1)(X3 +
X4- 1)(X 3 +X 2 4- 1).
The polynomial X 4 4- X + 1 is irreducible in Z 2 [X] so the quotient Z 2 [X]/
(X 4 4- X + 1) is a field with 16 elements; for it can be regarded as
{a + bx + cx2 +dx3 \ a,b,c, de Z 2 ,x 4 =x 4-1}.
The powers of X in the image are
X,X 2 ,X 3 ,X+1,X 2 4-X,X 3 +X 2 ,X 3 +X4-1,
X2 + 1, X3 4- X, X2 4- X 4- 1, X3 4- X2 4- X,
X 3 +X 2 4-X+1,X 3 4-X 2 + 1,X3 + 1,1.

86
Solutions to Chapter 2

These elements exhaust the non-zero elements of the field.


However, in the quotient Z 2 [X]/(X 4 + X 3 + X2 + X + 1) the powers of
Xare
X,X2,X3,X3+X2+X+1, 1
which generate a proper subgroup of the multiplicative group. This multi-
plicative group is cyclic, however, since X + 1 is a generator.

87
Test paper 1

Time allowed: 3 hours


(Allocate 20 marks for each question.)
1 If G is a group then x G G is said to be a commutator, if for some a,bGG,
x = [a,b] = a~lb~lab.
Determine the set H of commutators in S3 and show that it is a normal
subgroup of S3.
Prove that K is a normal subgroup of a group G if and only if [k, g] G K
for every kGK and everygGG.
2 Let G be the multiplicative group of non-singular n x n matrices with
rational entries. Let / : Sn -> G be defined as follows : for every permutation
Trlet
f1
/ O ) = [fl//] where fl/y = {
10 if
Prove that / is a group morphism and that K e r / = {1}. If IT = (k /) is a trans-
position prove that det f(n) = —1. Hence show that no permutation nESn
which can be written as a product of an even number of transpositions may
be written as a product of an odd number of transpositions.
3 Show that
H = {(1), (12)(34), (13)(24), (14)(23)}
is a normal subgroup of 5 4 . Let K consist of those elements of £4 which
leave 4 fixed. Prove that K^S3 and that H n K = {1}.
Show that if hi9 h2 EH and kh k2 GK then hxki= h2k2 implies ht = h2
and k\ = k2. Deduce that any element TTG&I. can be written uniquely as
Test paper 1

TT = hk where hEH and k€K. Use this fact to construct a surjective mor-
phism from £4 to K, and deduce that 5 4 has both a subgroup and a quotient
group isomorphic to S3.
Let p be a fixed prime and let i? be the set of all rationals expressible in the
form m/pk where m, k E Z. Prove that R is a subring of the ring (EL Is JR an
integral domain? DoesR contain a 1?
Find all the invertible elements of R and show that R is not a field.
For every a GR define aR = {ax | x GR}. Prove that aR is a two-sided
ideal of /?. Suppose now that q is a prime with (7 =£ p. Show that #/? is
a proper subset of jR whereas pR coincides withi?.
Show finally that if £>, #i, #2 a r e distinct primes then

5 Let A GMatnxw((Q) and let i%y be the matrix whose only non-zero element
is a 1 in the (i,/)th position. Calculate EimAErs.
Let / be a non-zero ideal of the ring Matwxw((Q). Deduce from the above

Show that # : Mat 2 x 2 (^) •* Mat3x3((Q) defined by


—c c c +d

4: ai- —a a a + b
—c c c+d
is a ring morphism. Deduce that
—c c c+d
S= —a a a + b \a,b,c9dG(H
—c c c+d
is a subring of Mat 3x3 (Q) that is isomorphic to Mat2x2((D). What is the
identity element of 5?

89
Test paper 2

Time allowed: 3 hours


(Allocate 20 marks for each question.)
Let (D be the additive group of rationals. Consider the group G = (Q/Z. Show
that G is infinite. Show also that every element of G has finite order.
Show that, for every integer n > 2, G has an element of order n. Show
also that any finite collection of elements of order n lie in a cyclic subgroup
of order n.
Let G be the group of 2 x 2 matrices of determinant 1 with entries in Z 3 .
Show that if

-G
then x, yGG. Prove that x 4 = 1, x2 — y2 and xyx =y. Deduce that the
smallest subgroup H of G containing x and y has order 8. Prove that H is
not abelian but that every subgroup of //is normal in//.
Let G be a finite group with an isomorphism $ : G^G satisfying # 2 = id^
and the property that if g^ 1 then &(g) ¥"g. Show, by multiplying on the
left by [tfO)]" 1 a n d o n t n e right by a, that
iHflV"1 = H^b'1 implies a = Z>.
Deduce that if g^G then g = &(a)a~l for some c G G and show that
&(g) = g'1. Prove that G is an abelian group of odd order.
Let mh m2 be fixed positive integers which are coprime. Show that for any
given integers ah a2 there exists an integer y such that
ai + ym\ = a2 (mod m2)

90
Test paper 2

and deduce that the simultaneous congruences


x = ax (mod m^), x=a2 (mod m2)
have a solution. Show moreover that this solution is unique modulo
Let R = Z mi x Z m2 with the ring operations
(a,5) + (c, J) = (fl + c , H d),

where a, cGZ m i and b, dGZ m 2 . Prove that R is isomorphic to the ring

5 Show that if / and / are ideals of a ring R then / + / = {z +/' | z £ / , / £ / }


and / n / are ideals of JR.
For every aG <Q let / a = {/G <h[X] \ f(a) = 0}. Show that Ia is an ideal
of (D[X]. Show also that if a=£j3GQ then every element of (D[X] can be
expressed in the form (X-a)h(X) + (X-p)g(X) for some h(X\
g(X)e<b[X].
Show that the assignment

defines an isomorphism from the ring (D [X]/Ip to the ring Ia/(Ia

91
Test paper 3

Time allowed: 3 hours


(Allocate 20 marks for each question.)
1 Let p be a prime and let G be the set of triples (x, y, z) of integers modulo
p. Define a multiplication on G by

Prove that G is a group of order p3. Show also that G is not abelian and that
the set H of elements of the form (x, y, z) is an abelian subgroup of order p2.
2 Find the eight elements of the group G generated by the permutations
a = (1234), Z> = (24).
Show that H= {1, b) is a subgroup of G, and determine the left and right
cosets of H in G. Is / / a normal subgroup of G? Show also that 1 and a2 are
the only elements that commute with every element of G.
3 Let A be a ring. For every non-zero k^A let
Sk = {ka\aGA}.
Prove that 5^ is a subring of A Show also that if A has an identity element
and if k has a multiplicative inverse in A then 5^ coincides with A.
Determine all the sub rings of the ring of integers modulo 6.
4 Let D be an algebraic system that satisfies all the axioms for a ring except
perhaps for commutativity of addition. By expanding (a + b)(c + d) in two
ways using the distributive laws, show that
ad + be = be + ad.
Hence show that, if D contains an element / that is right cancellable for
multiplication (in the sense that if gf—hf then g = ft), then D is in fact
a ring.

92
Test paper 3

Suppose that F is a field consisting of four elements a9 b9 c, d such that


a + b = b and be = c. Show that there is only one possible Cayley table
for multiplication, and find it. Deduce that c(d + d) = b + b. Hence deduce
that there is only one possible Cayley table for addition, and find it.

93
Test paper 4

Time allowed: 3 hours


(Allocate 20 marks for each question.)
1 Prove that
ffcoshjc sinhxi
G = \\ IJCGC
IL sinh x cosh x J
is a group under multiplication. Prove also that this group is isomorphic to
the additive group (D. Deduce that every finite subset of G is contained in
a cyclic subgroup of G.
2 Let # = { 1 , 2, 3, 4, 5, 6}. By considering decompositions of permutations
into disjoint cycles, or otherwise, determine all the permutations p on E such
that p2 = (135)(246). Show that there is no permutation p on E such that
P 2 =(14)(25)(36).
3 Let a : R -> S and ]3 : R^-T be surjective ring morphisms. Prove that the
following statements are equivalent:
(a) there is a ring morphism y : S -» T such y o a = j3;
(*) KeraCKer/3.
Show that every ideal of the ring Z is of the form riZ. for some « G Z .
Deduce that if R is an infinite ring and a : Z ->/? is a surjective ring mor-
phism then/? ^ Z.
4 Let A be a ring with identity element 1. Let aGAbe such that
(fl) a is not a zero divisor;
(b) QbeA)ab = l.
By considering the element aba—a, or otherwise, show that a is invertible
and that its inverse is b.

94
Test paper 4

Suppose now that A in finite and that xGA is neither zero nor a zero
divisor. If A = {xl9..., xn} show that the elements
xxi, xx2,.. .,xxn
are distinct, and deduce that xy — 1 for some j>EA Conclude that x is
invertible.
Let R be a ring. If there exists a least positive integer m such that
(\/aER)ma = 0 then /? is said to be of characteristic m\ and if no such
integer exists theni? is said to be of characteristic zero.
What are the characteristics of the rings Z and Z w ?
If R has an identity element 1, prove that R is of characteristic m if
and only if m is the least positive integer such that ml = 0. Prove that an
integral domain of characteristic non-zero is necessarily of characteristic
p for some prime p. Deduce that a field is either of characteristic zero or
of prime characteristic, and give an example of each.
If Fis a field of prime characteristic p prove that
(VJC, y G F) (x + y)p =

95

You might also like